TBR Bio1 Opt

Share Embed Donate


Short Description

Download TBR Bio1 Opt...

Description

$'edfidft'il Ndw6

dv

.,,',

ai,..Ifg.Nuffi$;,s,ydrc t; tffpes'ofnfissues ,;'1,,'

."t'l'tr;'Mefi$ biil ,:' i.'

1

5;';Atttoa'PtteniHs

1.

".,

tt"ii.-"t,'' r;: ;. "" : "; .

4;,. The, Ne,ufomuSeillai.Iunetioni I l

ff

5:'J E $lt

"' ;=, o*po*ng,'.,

fipt€1"',,;,,r,:,.'.",:

,6;,ffi#bns:sy i';c nti,btff Bb qiffi5r.,;i',

us€

1lr,..

I

8..&eeeptCI. d'gen$orX,U gt,ri'

Pd

:::

:

:

cCi FACCagCs er

m R:;i

Speciali#lH

l{ '

E I M' *tI

'Ti'

e.'.

ffi,;P#paration

l

nnswet$l'

Biotogy

Nerve & Muscle

Types Of Tissues

Types Of Tissues Let's consider a series of discussions on cellular physiology. For example, we will consider how muscle and nerve cells function. How does the chemical energy of

ATP (which was generated in glycoiysis, the Krebs cvcle, and oxidative phosphorylation) become converted into the mechanical movement of, say, muscle cells? How is it that the chemical energy of ATP is converted into an electrical signal that al1ows various nerves to communicate with those muscles? Before we can discuss the cellular mechanisms of muscles and nerves, we first need to consider some of the general characteristics of cells, tissues, and organs. The general body plan of an animal is fairly simple and can be divided into a number of systems that represent a variety of organs working in concert with one another. For example, one body system you are probably quite familiar with is the skeletal system. Another is the muscular system. Others are the circulatory, integumentary (skin), endocrine, nervous, and digestive systems, to name but a

few. The digestive system is formed by an alimentary canal (gastrointestinal "tube") that begins at the mouth and ends at the anus. This system is suspended within a body cavity re-ferred to as the coelom. The coelom is separated into a thoracic cavity (upper) and an abdominal cavity (lower). These two cavities are separated by the dome-shaped mass of skeletal muscle called the diaphragm. Within the thoracic cavity, one finds the lungs and the heart. The abdominal cavity contains the liver, stomach, and iatestines.

As we examine

l"hese

various systems, we will find different levels of

organization. There are individual cells, and then there are cells of a particular type which coalesce to form tissue. One example of a tissue is the layer of epithelial cells that line one of the principal organs of the alimentary canal, the

stomach. some of the simple epithelial cells within the siomach secrete hydrochloric acid (pH = 1) to aid in the digestion of food. Other epithelial cells of the stomach secrete mucus to help prevent that acid from digesting the lining of the stomach. Still other epithelial cells secrete enzymes. These epithelial cells are just one type of tissue that is involved in forming the stomach. The stomach is also composed of other types of tissue. For example, nervous tissue helps to innervate the stomach, connective tissue helps to hold the stomach in its proper position, and muscle tissue helps to propel food through the stomach. Thus, these four groups of primary tissue (epitheliai, connective, muscle, and nerve) have the abiiity to form the various organs of the body. An organ is n structure that is composed of two 0r ffLlre tissues that act in such a way as to perform a specific .function.

Epithelial Tissues ce11s in a little more detail. The epithelial tissue that constitutes the various organs of the body can be either simple epithelium (consisting of a single layer of cells) or stratified epithelium (consisting of two or more layers of cells). These epithelial celtrs come in a variety of shapes and sizes. For example, there are squamous (flat), cuboidal, and columnar epithelial cells (refer to Figure 1-1).

Let's examine the epithelial

Copyright @ by The Berkeley Review

i'

The Berkeley Review Specializing in MCAT Preparation

Biology

Nerve & Muscle

Types Of Tissues

On the lumenal side of the simple epithelial cells are projections cailed microvilli (singular, microviilus--see Figure 1-1 and Figure 1-2). These projections increase the total absorptive area of the cell (sometimes by as much as 25%). Sometimes you find specialized structures cailed cilia (singular, cilium) projecting outward on the apical surface of these cells. For example, in the respiratory tract these hair-like appendages move in a coordinated unidirectional wave to move foreign particles out of the mucous lining of the lungs and bronchial tubes.

Simple squamous epithelial cel1s

Simple columnar epithelial cells

\-J

Basal .--------\ lamina

Cuboidal and columnar epithelial cells

Stratified squamous epithelial cells (non-keratinized)

Figure l - l Types of epithelial cells.

Thebe cells are !_l-undea by a number of specialized junctions. For example, tight junctionS aet-fs a permeability barrier (see Figure 1-2). Not oniy do they prevent

the transport of protein molecules from the lumenal side of the cell towards the basolaterai side of the cell, but they also act to hold neighboring cells together.

Epithelial cells are also held together by structures called desmosomes (see Figure 1-2) One type of desmosome joins the epithelial cell to a structure on the basal side of the ceil called the basal lamina (or basement membrane). The basal iamina is in close contact with connective tissue that helps to anchor the cells in place.

Gap junctions provide a means for water-soluble molecules to pass from the cytoplasm of one cel1 to the cytoplasm of another cell (see Figure 1-2). These lunclions allow for equilibration within the connected epithelial celis and therefore allow those celis to function as a unit. For example, the beating cilia appear to be coordinated by waves of calcium, which flow in the plane of the juxtaposed epithelial cells.

Copyright @ by The BerkeleY Review

4

The BerkeleY Keview Specializing in MCAT Preparation

Biology

Nerve & Muscle

Types Of Tissues

LUMEN Apical surface

€ (=l

Tight junctions

Microvillus Tightjunctions

Gap

.lunctlon Desmosome Desmosomes Basal lateral surface

C) €f

BLOOD

Basal iamina (basement membrane)

Figure 1.2 Different components of an epithelial cell.

As we have mentioned, epithelial cells can secrete substances into a lumenal space. For example, hydrochloric acid can be secreted into the lumen of the stomach. If a cell secretes a substance into the lumen by way of a duct, it is referred to as an exocrine gland. Endocrine glands secrete substances into the blood. For example, insulin is a protein hormone secreted into the blood by clusters of specialized epithelial cells in the pancreas. Dead skin cells

&r \t-

(keratinizedl

I I noio..-ur f cells

)

Basement membrane

Collagen fibers

] conn..,,u. tissue J

t)

Figure 1.3 Stratified squamous epithelial

ce1ls.

Stratified squamous epithelium usualiy has a protective function. Your skin is composed of many layers of stratified squamous epithelial cells. The outer cells of your skin are dead, and they contain a large amount of the fibrous protein keratin (Figure 1-3). These cells are constantly being lost and replaced, as cells begin to move toward the surface from beiow.

Copyright

@

by The Berkeley Review

The Berkeley Keview Specializing in MCAT Preparation

Biology

Nerve Er Muscle

Types Of fissues

Consider a segment of skin. This organ comprises about 15o,i, of r-our total body weight. The epidermal region contains stratified epithelial cells that act to protect the deeper layers of the skin. Belou'the epidermis is the dermis. Within the dermis are a variety of structures. Surrounding the hair follicles are erector muscles, which act to straighten the hair shaft. This causes the skrn close to the hair follicle to become depressed and gives the characteristic appearance of "goose pimples." Those erector muscles are innervated biz nerves which cause them to contract at specific times (e.g., when it is cold outside). The skin is also a highiy vascularized organ. When it is hot outside, the blood is shunted towards the surface of the skin where it can dissipate some of its heat to the outside environment. Below the dermis is the subcutaneous tissue. This is where one

finds adipose deposits.

Connective fissues This type of tissue helps to anchor and support the various structures of the body. There are a variety of types of connective tissues, a few of which are structural, blood cells, mast cells, adipose cells, and melanocytes. Many of the proteins that make up structural connective tissue are secreted by cells cailed fibroblasts. Collagen, reticulirl and elastin are structural proteins which are secreted by these cells. Collagen is a triple-stranded, insoluble, fibrous protein (see Figure 1-3) that is highly ooss-linked, a feature that makes these fibers quite strong and rather flexible. Besides having a very high tensile strength, collagen is also the most abundant protein found in mammals. Reticulin is a thin fiber found in the spleen and lymph nodes. It is not as highly coiled as collagen. Elastin is also a highly cross-linked protein found associated with organs that require some degree of elasticity (like the lungs, skin, and blood vesseis).

Another type of structural connective tissue, cartilage, is secreted by a specialized fibrobiast cell called a chondrocyte. There are different types of cartilage, but in general it is found in places where there is a certain amount of stress placed on the body. For example, cartilage can be found in the nose, on the articulating surfaces of bones (including the intervertebral discs of the vertebrai column), and in the external ear.

Bone is also a structural connective tissue. About one-third of the weight of bone comes from organic materiai such as collagen, while the remaining two-thirds is inorganic material such as calcium phosphate and calcium carbonate. The collagen found in bone matrix is secreted by specialized fibroblast cells called osteoblasts. Collagen lends flexibility to bone, while the inorganic crystals lend rigidity. Within the centrai cavity of bone, we find a spongy marrow where red blood cells and white blood cells are formed. Towards the surface of bone the

ceilular arrangement is more compact. [As a comparison, the main structural component of chitin (found in the exoskeleton of insects) consists of specially modified glucose residues linked to one another to form long polymers. Associated with these polymers is calcium carbonate (CaCO3). This combrnation adds rigidity to the exoskeleton, but offers little in the way of flexibility.l We mentioned that blood cells and mast cells are kinds of connective tissue. We discuss blood cells in a separate lecture. Mast cells can be found in the respiratory tract, as well as in the gastrointestinal tract. Mast cells can release

will

Copyright

@

by The Berkeley Review

6

The Berkeley Review Specializing in MCAT Preparation

Biology

Nerve 8r Muscle

TYpes Of Tissues

histamines in response to an allergic reaction, an infection, or even an injury. Histamine causes an increase in blood flow to the blood vessels of the affected region.

Other types of connective tissue involve adipose cells and melanocytes. Adipose cells are simply ceils that store fat whereas melanocytes are cells which store pigments.

Muscle fissues will be discussing various types of muscie in future lectures. For example, r,vhen we examine skeletal muscle we will find that it is voluntary muscle. That

We

is, we can generally control its action. Cardiac muscle and smooth muscle are examples of involuntary muscles.

Nervous fissues The nervous systems allow one to adapt rather quickly to external stimuli. For example, consider a simple reflex arc. If someone were to tap on your knee with a

rubber hammer, your lower leg would extend outward. As the hammer impinged upon the patellar tendon in your knee, an eiectrical impuise was generated and traveled via a sensory nerve to your spinal cord. That sensory neuron svnapsed with a motor neuron, which returned the impulse to the muscle that was initialiy stimulated and caused it to contract. We will come back io this exampie and examine it in a bit more detail later. First, let's consider some terrninology.

Neurotransmitters

Dendrites

are released from sYnaPtic Uulbs

--i

A Typical Neuron Ceil body

Figure I-4

1r

:nrjor components of a neuron.

associated supporting cells make up the newous system. Nerve are also called neurons, and ihey are the basic structural unit that make up --:r: r.ervous system. The major anatomical features of a neuron are the cell body

\en-e cellsand :=

-s

-:.-,-oh'ed in integration of information), the dendrites (involved in receiving and ::a:.smitting information towards the cel1 body), and the axon (involved in :,:.ducting information away from the cell body). When a neuron becomes :\.-ri€d and receives electrical information in the form of a stimulus, the celi body ::it.esses that information and transmits it down the axon in the form of a nerve rnpuise called an action potential. When that action potential reaches the end*o{ -j-.. a\on (referred to as the synaptic bulb or bouton terminal), it causes the

,,::" nght O by The Berkeley Review

The BerkeleY Review Specializing in MCAT Preparation

Biology

Nerve & Muscle

Types OfTissues

release of a chemical substance called a neurotransmitter (see Figure 1-4). The neurotransmitter diffuses across the synaptic cleft and induces an identical action potential in an adjoining neuron, muscle cell, or gland cell. The junction between two such cells is called a synapse.

(

Copyright

@

by The Berkeley Review

a

The Berkeley Keview Specializing in MCAT Preparation

Biology

Nerve & Muscle

Membrane Potentials

The generation of electrical signals in the nervous system is concerned with the diffusion of ions from a high to a low concentration (see Figure 1-5). in other words, charged ions diffuse down their concentration gradient. In the extracellular space of vertebrates the concentration of Nao is about 150 mM, while that of Ko is about 5 mM. The concentration of Cle is about 130 mM, while ihat of HCO3e is about 25 mM. Note that the concentrations of the cations (Nae and KCI) and the anions (Cl€ and HCO3O) balance one another. In other words, we find 155 mM of the cations and 155 mM of the anions. This represents

electroneutrality. Outside Cell

Inside Cell

K* =

120 mM

to

140

K+=5mM

mM

Na* = 10 mM to 15 mM

]..,,"",

Na* = 150 mM

Cl-=5mMto40mM

Cl- = 130 mM

HCO3- = 12 mM to 25 mM

HCO3- = 25 mM

-

Proteins

Figure I -5 I. :.::r ceiiular

l^","",

concentrations of the common ions.

Ke (about 120 mM to 140 mM) =,:. a lorv concentration of Nao (about 10 mM to 15 mM). We also find a lower : -:.::nfation of Cle (about 5 mM to 40 mN! and usually a lower concentration "

r--r'n the cel1, we find a high concentration of

: :-rCO3 3 (about 12 mM to 25 mM). There are many negativeiy charged proteins ,' :--:ir. the cell. Electroneutrality can also be found on the inside of the cell as

:

-.

the distribution of ions across the cell's membrane, you will find it - r: :.::.-mrnetri.cal. Let's consider a resting nerve (i.e., a nerve that is waiting to :: .=:;-:: an action potential) that has a permeabiiity to potassium which is much .r:-:i:r :han its permeabiiity to sodium. In other words, PKe >>> PNao (where F :=:::s to permeability). Because the concentration of Ke is higher inside the ,: --r.;r: outside the cell, potassium will diffuse down its concentration gradient .: : -.:'" e 'Jee ceil (see Figure 1-6). As the positive Ko cation leaves the cell, there : -::::>:cndrngiy less positive charge remaining inside the cell. In other words, -- = ,: s:ie oi the cell is now more negatively charged with respect to the outside . ---- ::--. This sets up a voltage that is positive on the side of the celi to which ::::::-'':. is trving to diffuse to (i.e., the outside of the celi). As that positive :-,:i- ::gins to build up, it tends to push potassium back into the cell :;::::,:.:. irke charges repel each other).

:

. - .. -ook at

I

The Berkeley Keview Specializing in MCAT Preparation

Biology

Nerve & Fluscle

Membrane Potentials

Those tn'o forces (chemical and electrical) do not exactly match each other. It lums out that the force of diffusion is a little larger than the electrical force. This results in a little bit of leakage of Ko out of the cell, as well as a little bit of Ieakage of Nao into the cell. The Ke that leaked out of the cell has to be pumped back into the cell, and the Nao that leaked into the cell has to be pumped out of the cell. The pumping action of these two ions is provided for by the Nao/K@ ATPase pump. This pump is responsible for the generation of the asymmetrical concentration gradient of Nao and Ke across the cell's membrane. Cell membrane

Diffusion

Inside cell

Outside cell

Where P* ))) P"u

Na+

-87mV

+ 87

mV

Na*

and the anions are

impermeable

Figure l-6 Cellular gradients where PK >>> PNa.

We can calculate the membrane voltage (the potential difference) across the cell's membrane using the Nernst equation as shown in (L-1). In this equation, V is the voltage in miilivolts (1 mV = 10-3 volts), i refers to inside, o refers to outside, R is the gas constant, T is the temperature in Kelvin, Z is the ion's valance, and F is the Faraday constant. If we let the cell's membrane be permeabie to just Ko, we find that the voitage is '87 mV inside the cell with respect to outside the cell. Remember, this is if potassium is the only permeable ion. It is ihe gradient of potassium alone across the cell's membrane that is able to generate this potential.

[K*]o V'^ = 2.3 RL ,on " ZF [K*]r

(t-1)

[5 mM]" [140 mM]r

(1-2)

Vio = 60 log

Vio=-87mV

Copyright

@

by The Berkeley Review

(1-3)

lo

The Berkeley Review Specializing in MCAT Preparation

Biology

Nerve & Muscle

Membrane Potentials

a nerve, it leaves its resting state and enters an active state in r,r'hich the cell's membrane is more permeable to sodium (Nuo) than it is to potassiurn. In other words, PNa >> PK. Since the concentration of sodium is higher outside the ce1l than inside the cel1, sodium will tend to diffuse down its concentration gradient and into the cell.

If we stimulate

Once again, we can establish a separation of charge. As the Nao ions enter the ce11 ivith their positive charges, there is that much iess positive charge outside the .e11. The outside of the cell becomes more negativeiy charged than it was before \a3 started to diffuse into the cell. Similarly, as the Nao ions diffuse into the -e11, the inside of the celi begir-rs to accumulate more positive charge. The inside :- rhe cell becomes more positively charged than it was before the Nao ions :t.:ied to diffuse in (see Figure 1-7). As the Nao ions diffuse into the cell down --:-:ir concentration gradient, a chemical and an electrical equilibrium is being

=r:a:rished.

Inside cell

Where P*, )) F

Outside cell

" Na+ Voltage

:x,qLu'e l. / : . - -r :-:- :1::. 'ihere P\a >> PK

: -:-- -:-,:-:ie:he magnitude of the potential across the ceil's membrane by -- - - : I\-:llL,8ffi=%Quyosin Z-line

H-zone

Sarcomere

Figure

H-zone

Z-line

Sarcomere

I - 16

Sarcomere details

Myosin fiiaments are arranged toward the center of the sarcomere. They give rise to the characteristic dark bands one sees when examining muscle tissue. Actin filaments are attached to the Z-lines. The actin and myosin filaments interact with each other through projections stemming from the myosin fiiaments. Those projections are sometimes referred to as myosin heads. The myosin thick filament does not have any of these head groups in its central region but rather concentrates those head groups towards its terminal regions. The actin thin filament is composed of a protein subunit called G actin ("G for giobular), which is roughly spherical in shape. The actin filaments can grow by the addition of G actin to the ends of the already existing filament. Each actin

fiiament is composed of two rows of G actin monomers wound around each other to form a helix.

Actin and Myosin Let's consider how myosin and actin interact with each other at the molecular level. When a muscle is in its relaxed state, ATP is bound to the myosin head groups. The myosin head is not bound to the actin filament, because ATP reduces myosin's affinity for actin. \,Vhen ATP is bound to the myosin head, the myosin head itself is at about a 45' angle with respect to the actin filament. Since actin is not bound to myosin, the ATP on the myosin heads is hydrolyzed to ADP and Pi (inorganic phosphate). These hydrolysis products remain on the

myosin head. More importantiy, the myosin head now undergoes a conformational change, so that it is situated at a 90' angle with respect to the actin filament. This high energy, stable, myosin'ADP-P1 head complex binds to the actin filament. As we will see, this step is dependent on Ca2e being present. Copyright O by The Berkeley Review

22

The Berkeley Review Specializing in MCAT Preparation

Biology

Nerve & Muscle

The Sarcomere

The interaction between the actin filaments and myosin head groups causes the reiease of Pi and then ADP from the myosin heads. This process causes a conformational change iir the myosin heads, so that they shift by about 45' in a direction that is away from the Z-lines. This step, in which the actin filaments move relative to the myosin filaments, is cailed the power stroke, and the product of this step is referred to as the rigor complex or rigor state. See Figure 1-17.

ATP binds and causes myosin head

Mvosin

This is the rigor complex

r

ADP

f-ilament

:ioVes

Actin and myosin bind

rxgure l.I/ -'

a

:

-l::

-:Lrn Cf'Cle.

:-i.:: --:.. :TL\-osirl heads have swiveled and have pulled the actin filament past : : '- : :-rr, tilament, the myosin heads remain bound to the actin filaments. This

'-"

:-:

:=:..:-:alied rigor state. In order for the myosin heads to dissociate from the

i::*- 1ir1ents, ATP needs to bind to those myosin

heads. Remember, ATP

'.:,::s:r-.-osin's affinity for actin. Without ATP, muscles remain in a state of .".-:-: -:: : gir-en period of time. This is what happens to muscles in your body .-.-- -.----. :ie (i.e., rigormortis). The myosin head groups can no longer bind

:lT :=-;use the metabolic ",i = -=-..i to function.

pathways that generate this energy-rich nucleotide

lfi'u'oponin, Tropomyosin, Et Calcium r" r: : ': .'ra::Lln€d actin, we saw that it was composed of two long rows of

*::- , ;::.:rica1 protein subunits, which polymerized together and then wound : -r :;:: :--.Ler rn a helical fashion. If we look closely at the helical structure of r--:-i .:.:ii:e tn-o grooves. Running the length of these grooves is a coiled : *, ::*- :.--=j tropomyosin, composed of two helical polymers wound about --: =: :ee Figure 1-18). When tropomyosin resides in the actin groove, it -; - : :u:ding sites for the myosin heads and prevents those head groups -: -:':.1

:o the actin filament.

fhe Berkeley Review

The Berkeley Keview Specializing in MCAT Preparation

Biology

Nerve & Muscle

The Sarcomere

In the case of striated (skeletal) muscle, this is often referred to as the actin-based regulation of muscie contraction. Cardiac muscle and smooth muscle are both controlled by myosin-based regulation. Troponin Protien Complex

ca2+ Binding Tropom yosin

Site

o Relaxation

/t

"t llll f,l Calcium \1/

Removal

Addition ot Calcium

li

Contraction Myosin Head Binding Sites

Figure

l-lB

Sarcomere detail.

Troponin, a multi-subunit binding protein, interacts with tropomyosin, actin, and Ca2o. \A/hen Ca2o is bound to a particular subunit of the troponin complex, it causes tropomyosin to shift its position and expose the myosin head binding sites. The myosin heads then bind to the actin filaments (see Figure '1'-17) and muscle contraction follows. [In Figure L-17, Ca2@ would bind at Step 3.] lf Ca2o is not in the medium, there witl be none to bind to the troponin complex and tropomyosin wili remain in the actin gloves and cover the myosin head binding sites. This is the relaxed state.

Surrounding each myofibrii is a membranous structure,

a

modified version of the

endoplasmic reticulum called the sarcoplasmic reticulum. Calcium is sequestered within this smooth membranous structure. Also surrounding each mvofibril is an invaeination of the sarcoleuuna (i.e., the plasma membrane) called the transverse tubule (abbreviated as T-tubules). These T-tubules follow lhe Zlines of each myofibril. Selected anatomical features of the structures associated with the sarcoplasmic reticulum are shown in Figure 1-19. After an action potential crosses the iast synaptic junction on its way to a muscle cell, it passes down each T-tubule and somehow stimulates the release of Ca2e from the sarcoplasmic reticulum.' [At the present time, it is thought that the lumen of the T-tubules and the lumen of the sarcoplasmic reticulum are not

Copyright O by The Berkeley Review

24

The BerkeleY Review Specializing in MCAT Preparation

Biology

Nerve

8e

Muscle

The Sarcomere

continuous.] As the Ca2s flows from a region of high concentration (the sarcoplasmic reticulum) to a region of low concentration (the cytosol), it binds to

its binding site on the troponin complex. Each sarcomere

contracts simuitaneously because of the way in which the nerve impuise is carried along the sarcolemma and into the T-tubules.

+

Sarcolemma

Myofibrils

ransverse tubule

Figure l-19 -

-

=,.:,'oplasmic reticulum.

- ,- :-

:cntraction has taken place anC the nerve impulse has ceased, the Ca2e in --= :-.:osol is pumped back into the sarcoplasmic reticulum by a Ca2o-ATPase --::-: In the sarcoplasmic reticulum Ca2o associates with a specific binding ::,:: :.. \Vhen the next action potential passes down the T-tubuies, the cycle will -:.larn and another muscie contraction will take place. :.:e a number of ways ir-r which the strength of muscle contraction can be The strength of contraction can be varied by (1) the size of the motor unit :efined in a moment), (2) the number of available motor units, and (3) the ': of acth and myosin contained with each cell.

tor unit is simply a motor neuron and the muscle fibers that it innervates. .. e already examined the muscle fibers. What is a motor neuron? Motor . :-s are nerve cel1s whose ceII bodies are located in the centrai nervous :- e.g., the spinal cord or brain stem), and whose myelinated axons :,e skeletal muscle. Reca11 that myelinated axons allow action potentials to ::-,-::-,itted rapidly to the desired effector organ (in this case, a muscle) ,, anted to precisely move a muscle (e.g., the muscles associated with your - :ngers), then you wouid need motor units which were rather small in -:r smalier the size of the motor unit, the smaller the strength of . : ; ::- rn, In contraSt the posturai muscles of the back or the legs require large - . : .-::.:ts to be effective. Not only can strength be controlled but precrsion can

. : .--led as wel1. ::. :

by The Berkeley Review

25

The Berkeley Review Specializing in MCAT Preparation

Biology

Nerve 6t Muscle

The Sarcomere

The number of motor units also controls the strength of contraction. For example,

if you wanted to pick up a light object (e.9., u pencil), you would need to employ only a few motor units. However, if you wanted to pick up heavier objects (e.g., gym weights), you would need to utilize more motor units. When you lift weights regularly, the size of each muscle cell increases, because the amount of actin and myosin in each muscle cell has increased. The more actin and myosin in a muscle cell, the greater the

strength o{ contraction that can be generated.

Even though the strength of muscle contraction can vary with the size of the motor units, the number of motor units, and the amount of actin and myosin in each muscie cell, the ultimate determinant of muscie contraction is the concentration of ATP in your muscle cells. The energy for contraction comes from the hydrolysis of ATP to ADP and Pi (inorganic phosphate). See equation (1-7\:

ATP

----a

ADP + P; + Energy

(r-7)

For muscie contraction to continue, ATP must be constantly regenerated. Recail

from basic biology that under aerobic conditions (when 02 is not limited) glucose will be oxidized to CO2 and H2O. Energy can be extracted (in the form of 36 molecules of ATP--if we use the glycerol-phosphate shuttle) during this catabolic process from glycolysis, the Krebs cycle, and oxidative phosphorylation/electron transport. See equation (1-8): Aerobic (Slow)

Glucose

CO2

+ H2O +

36 ATP

(1-8)

During anaerobic conditions (when 02 is limited), glucose is metabolized to lactate (the anion of lactic acid). Only 2 ATP molecules are extracted for the use of energy in this process. See equation (1-9). [This equation is not balanced.] Anaerobic

Glucose

(Fast)

Lactate

+ 2ATP

(1-e)

Even though the efficiency of ATP production is greater for aerobic metabolism than for anaerobic metabolism (36 ATPs compared to 2 ATPs), ATP can be produced much more quickly through anaerobic metabolism than through aerobic metabolism, owing to enzyme regulation in the glycolytic pathway.

During anaerobic metabolism, the concentration of lactic acid begins to increase. This means that the pH of the medium will decrease (i.e., become more acidic). One of the key regulatory enzymes in the glycolytic pathway cannot function weli below a certain pH value. This enzyme has some optimum pH range at whictr it functions and if the pH falls below (or above) that range, the enzyme is inJribitedQlycolysis comes to a halt and the ATP yield becomes rnsufficient to /iatry out ndlmal metabolic processes. In other words, fatigue sets in.

Copyright @ by The Berkeley Review

26

The Berkeley Review Specializing in MCAT Preparation

Biology

Nerve Ef Muscle

ti$,#$,ft€

N.€

Nervous System Components i,,,.'r,,".,,,,' ,..

t,

..,.,.

The nervous system in its simplest form can be found among the cnidarians. All the neurons in these creatures are linked together in a nerve net (much like a rr.eb). stimulation of the nerve net causes muscle contraction. This simple procedure is referred to as a reflex arc. In the case of the cnidarians there is no

associative activity, meaning that the transmission of the action potential is not linfluenced by other neuronal activity.

-{ssociative neurons are {ound in higher animals (Platyhelminthes and above). Here, different neurons can interact to produce a given response. when a rrrlnber of these associative neurons are grouped together, the nervous system erpands. A grouping of nerve cells is called a ganglion (or a nucleus or nuclei). Groupings of neurons in higher animals also lead to more elaborate sense organs, :jtterentiation into a central region of cells (the central nervous system, ot -Ns; =ld a peripheral region of celis (the peripheral nervous system, or pNS), various :rssociative areas, and a brain. The three basic anatomicai divisions of the vertebrate brain can be seen in Figure 1-?0- Those divisions are: (1) the forebrain (prosencephalon), (2) the midbrain :resorcephaion), and (3) the hindbr ain (rhombencephalon).

Rhombencephalon

rfignr l-2o 'Mlmcai

Cirisions of the vertebrate brain.

erufurlcr icl rmeaning "below") the brain is the spinal cord. Recall that the brain ffiie spinaf cord together are the central neraous system (CNS). From the spinal ''rlrmfi r@!d* rrrmr-es extend into the limbs and extremities (the periphery) of the body. ffime mru1:es represent lhe peripheral neroous system (PNS). If a neuron carries

into the spinal cord and brain, that neuron is said to be an afferent i{lm*.'*yt nenron- U a neuron carries the information away from the brain or mnffimnnnmrnn#.ncne

qryilltumf rmmd-

that neuron is said to be an efferent (motor) neuron.

l[ffie Sunbrain has several anatomical subdivisions, including the cerebrum, md hrpothalamus (see Figure 1-21'.a). The cerebrum is divided into @{ilfril mr,{ lc{t cerebral hemispheres, joined by the corpus callosum. The cerebral lfirnlil'ilr1iql'ruttmsres are dirtded into the frontal lobes (associated with movement and Wmrmnlilittmy ri" parietal lobes (associated with touch and stretch sensation),

ilhihr

niltntldmdtll

rfti

ldes

nirilmtwr,,os

i{0fu@fh,@

(associated

shonrr in Figure

@,

with vision), and temporal lobes (associated with

'1,-21b.

Tbs Berkeley Review

27

The Berkeley Keview Specializing in MCAT Preparation

Biology

Nerve Er Muscle (a)

Nervous System Components (b)

Thalamus Cerebral

Central Sulcus

Cortex Corpus

Callosum

)/s /a

."o

\ Parjetal

o .)

Pons

The Cerebrum and its Four Lobes

\ Medulla

Spinal Cord

Cerebe llum

Figure l-2 I Anatomical divisions of the vertebrate brain.

The outermost layer of the cerebrum is called the cerebral cortex. It consists of gray matter, which is simply nerve cel1 bodies and their dendrites. Beneath the gray matter is the white matter, or myelinated axons of the nerve cells. In the spinal cord, the situation is reversed: The gray matter is more centralized, while the white matter is more peripheral. The cerebral cortex has many important landmarks, one being the central sulcus, a prominent groove that separates the frontal lobes and the parietal lobes.

Anterior to this sulcus is the motor cortex, which controls the movement of individual muscles. Posterior to this sulcus is the (somatic) sensory cortex, which detects sensations in various parts of the body.

Somatic receptors send their information into the spinal cord via afferent nerve fibers, which either cross over to the opposite side of the spinal cord and then ascend to the sensory cortex in the brain, or ascend in the spinal cord and then cross over in the brainstem before reaching the sensory cortex.

flomunculus Wilder Penfield, a Canadian neurosurgeon, was able to rnap the sensory and motor areas of the brain by electrically stimuiating certain regions of the brains of

his patients during sulgery and observing their actions. Throughout this procedure his patients were conscious but were unable to feel any pain because there are no sensory receptors for pain il the cerebrum. He was able to show that the largest number of cortical neurons found in the sensory cortex register sensation in the fingers, hands, lips, and tongue. This is represented as the sensory homunculus (a schematic model of a human being mapped out on the sensory cortex) shown in Figure 1-22. Penfield was also able to show that the largest number of cortical neurons found

in the motor cortex control individual movement of the fingers, hands,

and speech. Groups of muscles are controlled by an area just anterior to the motor cortex called the premotor cortex. This is an association area. Stemming from

2a

The BerkeleY Review Specializing in MCAT Preparation

Biotogy

Nerve & Muscle

Nervous System Components

this association area are neuronal connections with the thalamus and cerebellum. Together these structures plan specific movements that the motor cortex then executes. It is thought that cognitive functioning like speech, writing, and reading are localized in the left hemisphere of the brain, whiie intuitive functions are localized in the right hemisphere of the brain. This is not proven, only theoretical.

-l

-J

Sensory receptors on the right side of the body project to the somatosensory cortex on the left hemisphere of the brain and vice versa

Left

i

Hemisphere J ,

L ,

Tongue

Right Hemisphere

Figure l-22

l:.:

:.c,munculus.

.:.

thalamus is a relay station for much of the visual and auditory information -,\-e receive from our environment. The hypothalamus is concerned with the activities of the body. The pituitary gland is the master endocrine gland -':t:a1 ,,' -:.: body. It receives information from the hypothalamus and sends out ::::rahon to regulate different parts of the body. -*-::

'' = brainstem contains such anatomical feafures as the midbrain, cerebellum, : ::ls medulla, and the reticular formation. These areas coordinate motor and -.:=:;-- activities. The midbrain detects movement and can direct the head and : :!

:iir-drds that movement. The midbrain can also sense pleasure and pain. is responsible for the bulk of regulation and coordination of -: ..-.:-^iar activity. The pons and medulla coordinate visceral activities. The -. * ----:-:: formation, which is the core of the brainstem, is essentially an activating . :::::.;esigned to alert the brain. The reticular formation also inhibits motor i.- - ::.*ior:\- impulses and can induce sleep. Below the medulla is the spinal cord. --- = :e:ebel1um

br The Berkeley Review

The Berkeley Review Specializing in MCAT Preparation

Biology

Nerve & Muscle

Control Of Body Activity

G;dffitfdlrri drdyrfffifiiffii Recall that when we first mentioned the nervous system, we iooked at a simple reflex arc. If someone were to tap you on your knee with a rubber-headed hammer, your lower leg would extend outward. The mechanism behind this is quite simple. As the hammer impinges on the patellar tendon, sensory neurons located in the tendon of the quadriceps muscle are excited. These impuises travel along an axon that enters the spinal cord through the dorsal root gangiion and synapses with two neurons (Figure 1-23). This area, even though it is supposed to be gray matter, is shown as being clear so you can see the synapses. Biceps ('hamstring") muscie is a flexor

\*u.

I

Ganglion

Dorsal

Grav Matter

Root

Spinal Cord

Motor Nerve Polysynaptic Reflex Arc

Tibia

Interneuron

Figure l-23 The reflex arc.

One of the synapses is to a motor neuron that immediateiy leaves the spinal cord and returns to the quadriceps muscle, causing a contraction. As this muscle contracts, the leg straightens (extends) at the knee joint. Because it elicits this kind of action, the quadriceps muscle is termed an extensor muscle. The type of synapse just described (making just one synaptic connection), is referred to as a

monosynaptic reflex arc. The other synapse is made to an interneuron which, in this case, is inhibitory. This interneuron in turn synapses with a motor neuron that innervates the bicep ("hamstring") muscle in the back of the leg. When the bicep muscle contracts, the lower portion of the leg bends or flexes at the knee joint. We call this kind of muscle a flexor muscle, and this type of synapse (because it makes at least two synaptic connections) is referred to as a polysynaptic reflex arc.

If contraction of the biceps muscle is inhibited

as the quadriceps muscle

contracts, then one observes a smooth and coordinated movement at the knee joint, as the lower portion of the leg extends outward after stimulation by the tapping of the hammer on the patellar tendon.

Copyright @ by The Berkeley Review

50

The Berkeley Kevibw Specializing in MCAT Preparation

Biology

Nerve & Muscle

Control Of Body Activity

Neurovisceral Control The autonomic nervous system, which is part of the efferent division of the peripheral nervous system, can be subdivided into the sympathetic and the parasympathetic systems. Nerve fibers from the autonomic nervous system leave the spinal cord to irrrervate various glands, smooth muscle, and cardiac muscle. Let's examine these two subdivisions.

Paras;rrmpathetic Division The parasympathetic division of the autonomic nervous system has nerve fibers which leave from the sacral portion of the spinal cord and from the midbrain (mesencephalon) and medulla (part of the rhombencephalon), as shown in Figure L-24. Parasympathetic nerve impulses tend to increase the rate of digestion and lower the heart rate. The blood pressure is also iowered, and the pupils constrict (contract). In general, the parasympathetic division conserves energy and helps in the restoration of various bodily functions (".9., by aiding in

the digestion of food for later use in metabolic processes). Ganglion

Neurotransmitter is acetylcholine

Eve

Ganglion

1 Lacrimal glands

o

Postganglionic nerve fiber

\ur, \Itrl.a

lr

o (,)

I

Small intestines

Heart

Lungs and Bronchi

(tQ.i

\ \ Large intestines I

(

Sexuai organs

1

Figure l-24 Parasympathetic nerves.

The parasympathetic division has both preganglionic and postganglionic nerve fbers. The cell bodies of the preganglionic neurons are found in the sacral region Copyright

@

by The Berkeley Review

3l

The Berkeley Keview Specializing in MCAT Preparation

Biology

Nerve & Muscle

Control Of Body Activity

of the spinai cord and in the brainstem. The ganglia of the parasympathetic division lie near or in the organs that are to be innervated. Therefore, the preganglionic nerve fibers are rather long, while the postganglionic nerve fibers are rather short. Both the preganglionic and the postganglionic nerve fibers in the parasympathetic division release acetylcholine (A C h ) as the

neurotransmitter. INerve fibers that release acetylcholine as their neurotransmitter are called cholinergic nerve fibers.]

The most prominent nerve in the parasympathetic division is the vagus nerve [also called the (tenth) X cranial nerve]. Roughly three-quarters of all the neurons in the parasympathetic division can be found in the vagus nerve. The reason for this should become obvious, if you look at Figure 1-24 The vagus nerve innervates the heart, lungs, stomach, liver (not shown), small intestine, large intestine, and kidneys (not shown), among other organs.

Sympathetic Division The sympathetic division of the autonomic nervous system has nerve fibers branching off from the thoracic and lumbar regions of the spinal cord, as shown in Figure 1-25. Sympathetic nerve fibers tend to condition the body for a "fightor-flight" response (a term first proposed by the Harvard physiologist Walter Cannon in the early 1900s). The heart rate increases, blood pressure elevates, pupils dilate (open wider), and the digestive functions decrease, all as a result of sympathetic nervous innervation. The sympathetic nerves that leave the thoracic and lumbar regions of the spinal cord first enter chains of ganglia connected by nerve fibers on either side of the spinal column. These chains of ganglia are collectively called the sympathetic trunk. As these spinal nerves, called preganglionic nerve fibers, enter the sympathetic trunk, they can either (a) pass through this collection of ganglia to synapse with other ganglia outside the sympathetic trunk, (b) pass into the sympathetic trunk and ascend or descend to synapse with ganglia at other levels,

or (c) pass into the sympathetic trunk and directly

synapse

with a given

ganglion. The nerve fibers leaving a synapse in a given ganglion are referred to as postganglionic nerve fibers. In the case of the sympathetic division the preganglionic nerve fibers tend to be short, while the postganglionic nerve fibers tend to be longer. The preganglionic fibers release acetylcholine, while the postganglionic fibers release norepinephrine as their neurottansmitters. [Nerve fibers that release norepinephrine (or epinephrine (adrenaline)) are called adrenergic nerve fibers.l One set of spinal nerves that originates in the lower thoracic region of the spinal

cord send long preganglionic nerve fibers to the adrenal medull4 a region within the adrenal glands (located on the superior surface of the kidneys). These nerve fibers synapse directly on the ganglia in the adrenal medulla. There are no postganglionic nerve fibers. When the adrenal medulla is stimulated, both norepinephrine and epinephrine are released directly into the bloodstream. Because these substances are released into the bloodstream, we can refer to them as hormones. Therefore, the adrenal medulla can be considered as an endocrine

gland. These hormones, distributed throughout the body by the circulatory system, can increase the heart rate and cause the pupils to dilate.

Copyright @ by The Berkeley Review

32

The Berkeley Review Specializing in MCAT Preparation

Biology

Nerve & Muscle Postganglionic nerve fiber Eye

Control Of Body Activity

Neurotransmitters are acetylcholine and norepinephrine

{Vi,

t( .& LJ /^

Preganglionic nerve flber

Hearr

Thoracic Lungs and Bronchi

bo

Stomach O d)

Pancreas

o,

Small (n

Kidney

intestine

Large intestine

'o k O

aa

Bladder

Figure I -25 . -:.:::irC nefves.

Somatic vs Autonomic Nervous System

-: , : :evierv the basic differences between the somatic nervous svstem and

the

-: - ,-:r-ic nervous system. In the somatic nervous system, we find that (a) once . :.=:r,'e fibers ieave the central nervous system, they do not make a synapse :-- -:,ev have reached their effector organ. When the synapse is made at the -:--: organ, (b) the neurotransmiiter that is released is acetylcholine. The :

nervous system (c) innervates skeletal muscle. Innervation of that (d) Ieads to excitation of the muscle itseif. nuscle =:-':--:-:

autonomic nervous system, we find that (a) once the nerve fibers leave the

:::- :,en-ous system, they synapse with a ganglion before they make the final =:-= rvith their effector organ. The preganglionic fibers in both the ::':-rathetic and sympathetic divisions release (b) acetylcholine as the -:,-:=.smitter. The postganglionic fibers in the parasympathetic division =:'= a:eivicholine; in the sympathetic division, norepinephrine is released. ::nomic nervous system (c) innervates glands, and smooth and cardiac

l:'e

ce1ls innervated

by the autonomic nervous system can (d) be either

or inhibitory

The Berkelev Review

The Berkeley Review Specializing in MCAT Preparation

Biology

Nerve & Muscle

Control Of Body Activity

The adrenal medulla, a specialized ganglion in the sympathetic division of the autonomic nervous system, is directly stimulated by a preganglionic fiber. This nerve fiber releases acetylcholine, which causes the ceils of the adrenal meduila to release two types of hormones into the biood. The major hormone released is epinephrine. The minor hormone released into the blood is norepinephrine. These differences are illustrated in Figure 1-26.

CNS Skeletal muscle

Somatic Nervous System

Acetvlcholine Ganglion

Giands, cardiac and smooth muscle

Postganglionic nerve fiber

l-IE t'ts

Acetylcholine

Autonomic Nervous System

.*_|or-g".e

F+

Biood

(Epineprhine) Adrenal Medulla

Ganglion

Preganglionic nerve fiber

r-)

Norepinephrine

>s t4 l5 lo l-

)l6'

Organ

Glands, cardiac and smooth muscle

t la

t: IrJ ID J5

@

o

Figure l-26 CNS and PNS review.

Copyright @ by The Berkeley Review

34

The Berkeley Review Specializing in MCAT Preparation

Biology

Nerve & Muscle

KeGeptof5,:,and,ri .S€n5,6ff

Receptors and Sensory Input

,.,Iffit

There are many different types of receptors receiving sensory information from

the environment and passing that information to the nervous system. For erample, there are mechanoreceptors, which are concerned with pressure, hearlng, balance and blood pressure. Nociceptors sense pain. Mechanoreceptors :espond to a change in their configuration. Some can respond to a light pressure, -,r-hile others respond to a deeper pressure. There are specialized regions on the =:des of fish called lateral line organs that respond to a change in the pressure of :re rvater. There are pressure receptors in the walls of the aorta that are able to ::nse an increase in blood pressure.

Thermoreceptors detect cold and warmth, while chemoreceptors can detect -:--te, smell, oxygen, carbon dioxide, hydrogen ions, and blood giucose ievels. -:-.e taste receptors in the tongue can distinguish between food which is bitter, .:'.r, salty, or sweet. Olfactory cells rn the nasal cavity can distinguish hundreds - ilfferent odors. Photoreceptors in the retina of the eye can respond to the ::.sence of a single photon of light. There are also receptors concerned with :,ectricity and magnetism. For example, catfish have electrical receptors that ..=.r them detect prey, and birds have magnetic receptors that help them to ---:' 1gate. - -.= sensory information that a receptor receives is specific for that particular :=:E:tor. The stimulus that is received by that receptor changes (converts or ::ansduces) the potentiai (V*) across the receptor's membrane. This change in =::'.biane potential is called a receptor potential. If this receptor potential were ', ='.:eed a specific threshold, an action potential would be generated. We also :: -,rat if the pressure on this receptor is great and the change in membrane : :=:.iial is large, then the receptor potential will exceed the threshoid level. The :::*-: is an increase in the frequency of the action potentials being generated. . . = ,:nplitude of the action potentials r,vi11 not be change, only the frequency.

Receptors and Transducers .: . :onsider the receptor potential for a special type of mechanoreceptor ca11ed :acinian corpuscle. A pacinian corpuscle has an unmyelinated nerve ending . -::. is encapsulated in layers of connective tissue. However, the afferent nerve -- :, . -=ar-es

this encapsulated ending is myelinated. Saltatory Pressure

_/\:*v + +

\----l--\------r'

'','gUre 1.27

',, - .--. -'.rrpuscle.

.ii

0 by The Berkeley Review

cc

The Berkeley Review Specializing in MCAT Preparation

Biology

Nerve Et Muscle

Receptors and Sensory Input

ending is depressed, a local deformation causes sodium channels to open and Nao ions to rush into the nerve ending. An action potential is not generated in the receptor region. Instead, the establishment of this receptor potential causes a localized flow of current to be propagated along the nerve fiber. If the threshold has been reached, then this local current flow will jnitiate an action potential at the first node of Ranvier, located at the outer edge of the pacinian corpuscle. The action poiential can spread down the afferent nerve and towards the central nervous system by saltatory conduction, as shown in Figure \zVhen the nerve

7-27.

Suppose we were to depress the pacinian corpuscle just a smail amount. If the threshold is not reached, an action potentiai will not be generated. This means one would not feel the pressure that is being applied. If we apply a second stimulus, we might depolarize the membrane even more. If we were to apply a third stimulus that was even stronger, we could exceed threshold and an action potentiai would be generated. If we maintain this pressure such that we are just

above threshold by a certain amount, we will continue to generate action potentials at the rate of, say,2 action potentials every 10 milliseconds. 2 action potentials per 10 miliiseconds

3 action potentials per 10 milliseconds

Threshold

o lst Weak

[fr

3rd

Time (ms)

tl

Strength

of

stimulus

Strong

4rh

Figure l-28 Acti0n potential frequencies.

What wouid happen if we apply a fourth stimulus, much stronger than the previous stimuli? The membrane would become more depolarized, and we would move further above the threshoid level. Once this happens, the frequency of action potentials generated would increase, say, to 3 action potentials every 10 milliseconds. This is shown in Figure 1-28. Note that in each case the amplitude of the action potentials remains the same. \Alhat changes is the frequency of the action potentials propagating along the axon" It is the frequency of action potentials that signals to the central nervous system the magnitude of the stimulus being received. All receptors function on this basic principle.

Copyright @ by The Berkeley Review

36

The Berkeley Review Specializing in MCAT Preparation

Biology

Nerve 6r Muscle

Receptors and Sensory tnPut

Adaptation

wheriyou wake up in the morning and take a cool shower, you {eel the coolness of the water on your skin. When you get dressed, you feel the clothes touching your skin. In each case, though, after a brief period of time you get used to the water or the clothes touching your body. These are examples of sensory adaptation. One type of receptor that undergoes sensory adaptation is the pr"ir.r." receptor. if we plot the frequency of action potentiais (as they travel Lack to the central ,r*tno.tt system aiong a given axon) as a function of the time of a sustained stimulus, we would find that pressule receptors adapt rapidly' F{owever, pain receptors do not adapt raprdly, as is shown-in Figure 1-29' If the pressure ,u."ptott did not adapt to the touch of the clothes that we wear, it rvould plove to be rather inconvenient. [On the other hand, the body does not adapt ai readily to the sensation of pain--a good thing, evolutionarily speaking, silce pain is a warning of potentiai damage to the body's tissue and not something to be responded tb as a matter of convenience, but as a matter of sunival.]

)\= 'r O

Time of sustained stimulus (sec)

Figure l-29 -.:.::aiion to pressure

and to pain.

Tactile Discrimination

-"-=:J that the end of a neuron can be divided into many branches. Each of these ::::rches in turn can end at a receptor (such as a pacinian corpuscle). These 1.,--ra-l branches and their corresponding receptors constitute a receptive field. there can be many -.=:ending on which area of the body one is describing, :=,::iiit'e f-ieids, some of which overlap, or there can be a few receptive fields, :::-: o j tvhich do not overlap. Let's consider a set of receptive fields that overlap : :,rier io d.etermine how iwo points in space can be distinguished from one ci- :

same touch. =er bv the

:-::l=e \\e have a set of receptive fields like the ones shown in Figure 1-30a. ,--.., ."" generally finds is thit a greater frequency of action potentiais will be :

l 0 3

e

n .e

ra'

tn

if the central region rather than seem *: = :e:rphery of a glven receptive field is stimulated. In other words, there itsperiphery. in ,, , ," receptors in the central region of a receptive field than

r.:=:.:ed fass,tming threshold has been

reached)

-ore

outd happen if receptive fields overlapped, as shown in Figure 1-30? If ri: -.r-ere to sti.mulate the receptive field represented by (b) in Figure 1-30 ,:r:t:-i.*- enough, then because of the field overlap, we would also stimulate l::-= :::e::ive fields in (a) and (c). It would feel to a subject as if three different .r::. _: e body were being stimulated when, in fact, just the receptive field in from these b -. :eing stimulated. HJw can the information being received (b) is being field only .:5s.-r-. a-ierent neurons be fine-tuned to let us know that

r,,:::

-,,.

- :' :::::

3

b1' The BerkeleY

Review

5/

The BerkeleY Keview Specializing in MCAT PreParation

Biology

Nerve Ef Muscle

Receptors and Sensory Input

stimulated, and not field (a) or field (c)? This is accomplished by a process called lateral inhibition, mediated through interneurons within the spinal cord. In Figure 1-30, the axon that leads away from field (b) has lateral connections to interneurons that inhibit the impulses being sent down the axons from fields (a) and (c). In other words, because of lateral inhibition the impulses generated in receptive fields (a) and (c) are suppressed, allowing the impulses from the receptive field in (b) reach the proper spinal tracts that ascend to the brain. To Brain

6 a

(a)

+

(b)

D

(c)

+ Lateral Inhibition (interneuron)

Figure l-3O Receptive fields.

Somatic Sensory Pathways Once the sensory afferent nerve fibers enter the spinal cord they cross to the side opposite from the side they entered, either in the spinal cord or in the brainstem.

In other words, the sensory input from the right side of the body witl

be

represented on the somatosensory cortex of the left cerebral hemisphere, and sensory input from the left side of the body will be represented on the somatosensory cortex of the right cerebral hemisphere. [In Figure 1-39 which side of the body are the receptive fields located, the right side or the left side? Will the sensory input go to the somatosensory cortex of the right or left cerebral hemisphere? How do you know?]

Where do these sensory neurons synapse as they ascend to the brain? As a general rule, there are three neurons involved in sensory pathways. In the case of pressure, we find that the first-order neurons carrying information from the receptive field(s) enters the spinal cord and synapse with second-order neurons that ascend on the opposite side of the spinal cord to the thalamus. Here, another synapse is made with third-order neurons that continue to ascend, until they reach a specific region of the somatosensory cortex of the cerebral hemisphere opposite to the side of the body in which the sensation was perceived. The cerebral cortex itself contains ceils that are organized into 6 horizontal layers. The sensations of pressure would be registered in cells arranged in columns that cross these different lavers" Copyright @ by The Berkeley Review

3a

The Berkeley Review Specializing in MCAT Preparation

Nerrre & Musc[e To Go 15 Passages

98 Questions

Time for All Passages Taken Together as a Practice Exam 125 Minutes

I. il. IIII. IV. V. VI. VIII. WII. IX. X. XL XIII. XIIII. XIV. XV.

Passage Titles Types ofTransport Autonomic Nervons SYstem Action Potentials Local Anesthetic The Lens, the lris, & Associated Muscles Resting Membrane Potential Nicotine Replacement RetinalProjection Axonctl Transport Huntington's Disease Photoreceptors Sound Transmission in the Ear T-r,-ptophan & Serotonin Experiment Frog Muscle Experiment Skeletal Muscle Groups

,TftE

EIEru{EMY

L,g.p-y.1.fl-1ry'

Specializing in MCAT Preparation

Questions

1-8 9-1.3

14-20

2t-26 a l /./- - JJ

34-40

4r-46

47 -52

53-58 59-64 65-71 12-19 80-85 86-92 93-98

Suggestions The passages that follow are designed to get you to think irr a conceptual manner about the processes of physiology at the organismal ievel. If you have a solid foundation in physiology, *u1y of these answers will be straightforward. If you have not had a pleasant experience with the topic, some of these answers might appear to come from the void past the Oort field of the solar system. Pick a few passage topics at random. For these initial few passages, do not worry about the time. Just focus on what is expected of you. First, read the passage. Second, look at any diagrams, charts, or graphs. Third, read each question and the accompanying answers carefully. Fourth, answer the questions the best you can. Check the soiutions and see how you did" \A4rether you got the answers right or wrong/ it is important to read the explanations and see if you understand (and agree with) whal is berng explained. Keep a record of your results.

After you feel comfortable with the format of those initial few passages, pick another block of try them. Be aware that time is going to become important. Generally, you will have about

passages and 1

minute and 15 seconds to complete a question. Be a little more creative in how you approach this next

group. If you feel comfortable with the outline presented above, fine. If not, then try different

approaches to a passage. For example, you might feei well versed enough to read the questions first and then try to answer some of them, without ever having read the passage. Maybe yor.un answer some of the questions by just looking at the diagrams, charts, or graphs that are pr"r"rrtud in a particular passage. Remember, we are not clones of one another. You need to begin to develop a format that works best for you. Keeping a record of your results may be helpful.

The last block of passages might contain topics that are unfamiiiar to you. Find a place where the level of distraction is at a minimum. Get out your watch and time yontr!ff on these parruges, either individually or as a group. It is important to have a feei for time, and how much is passrng aslou try to answer each question. Never let a question get you flustered. If you carurot figure ont *nui the answer is from information given to you in the passage, or from your own knowledge-Lase, dump it and move on to the next question. As vou do this, make a note of that pesky question ind come back to it at the end, when you have more time. When you are finished, check your answers and make sure you understand the solutions. Be inquisitive. If you do not know the answer to something, look it up. The solution tends to stay with you longer. (For example, what is the Oort field?)

The estimated score conversions for 100 questions are shown below. At best, these are rough approximations and should be used only to give one a feel for which ballpark they are sitting in.

Section I Estimated Score Conversions Scaled Score

>12

l0-

11

8-9 7 6 5



::1. Descending

Aorta

Papillary

Bundle of flis gives rise to L. E( R. Bundle Branches (frrrther divides into Purkinje Fibers)

Muscle

ri.lre 2.5 - -.. -:ndmarks of the heart. -

-:

::.e heart there are valves between the right atrium and right ventricle valve), between the right ventricle and the pulmonary .:.. rulmonary valve, or tricuspid), between the left atrium and the left -:. :,= 'ie left atrioventricular valve, or mitral), and between the left ventricle -- .: r ird (the aortic valve). Once the ventricles are filied with blood and they - =:, :ontract the valves between the atria and the ventricles close. This : i: :r.\- backflow of blood into the atria and ensures that the blood will be : :-- rhe pulmonary and systemic systems of the body. The closing of the :.ricular valves between the atria and the ventricles as the ventricies are .- : ::1\'es the characteristic "lub" sound when listening to the heart. As ,: :-:rrs out of the pulmonary artery and the aorta, the pulmonary valve - :lrlic valve close, giving the characteristic "dub" sound. The closing of -. as blood is pumped either from the atria into the ventricles or from :-=s lo the puimonary or systemic tissues prevents backflow into either

: r l-- i atriorrentricular

fhe Berkeley Review

The Berkeley Review Speciatizing in MCAT Preparation

Biology

Ileart & Lungs

Cardiovascular Anatomy

the atria or the ventricles, respectivery. The valves between the atria and the ventricles themselves do not invert because of tendinous cords, called chordae tendineae, which hoid them in place.

Electrical Activity of the lleart

Located near the junction of the superior vena cava and the right atrium is a specialized region of myocardium called the sinoatrial node (sA node) or the pacemaker of the heart (Figure 2-4).

SA

AV Node

Bundle of His

Figure 2.4 The SA and AV nodes of rhe heart.

The SA node is the point of origin for the electrical impulse that propagates through the rest of the heart. Th[ electrical impulse spreads out over the atria causing them to contract and filt the ventricles. Located in the lower portion of the right atrium and near the right ventricle is the atrioventricular node (AV node). Impulses from the sA node also spread to the AV node and then from the AV node through a coliection of fibers called the bundle o{ His. Branches of the bundle of His surround the ventricles, and when this bundle receives an impuises it causes the ventricles to contract and eject blood to the pulmonary and systemic systems. Anatomicaily speaking, ventricular contraction is from the apex of the heart towards the base of the heart. It is interesting to note that if the sA node is damaged, the AV node takes over and slows the heart down to about 40 beats per minute.

Cardiovascul.rr Control The average blood pressure leaving the aorta is about 100 mmHg. This blood pressure is monitored by baroreceptors and chemoreceptors located in the carotid arteries and in the aortic arch. The baroreceptors (pressure receptors) are continually monitoring how much the aortic and carotid arteries are eipanding Copyright

@

by The Berkeley Review

8a

The Berkeley Review Specializing in MCAT Preparation

Biology

Ileart & Lungs

Cardiovascular Anatomy

and contracting. Suppose we were to decrease the arterial pressure. There would be less stretch on the aortic and carotid arteries. The baroieceptors would sense this and send impulses to the medulla in the brain stem. The medulla responds by activating the sympathetic nerves of the autonomic nervous system. Impulses are sent via the sympathetic nerves and norepinephrine is released at the sA node to increase the rate of the heart. This action helps to increase the contraction of the heart, which in turn wiil elevate the blood pressrrre. Sympathetic nerve fibers will also stimulate the adrenal medulla to releise epinephrine (adrenaline) into the blood. This hormone acts to increase the rate of the heart, therefore increasing the contraction of the heart. Both of these actions act in a negative feedback manner to "negate" the initial loss of pressure due toi say, hemorrhaging. The sympathetic nerves will also cause constriction of the blood vessels that lead to the gastrointestinal system and the kidneys. During hemorrhaging the brain and heart receive first priority in terms of blood. The rest of the organ systems of the body see a decrease in the flow of blood until the probiem is corrected.

Copyright

@

by The Berkeley Review

a9

The Berkeley Review Specializing in MCAT Preparation

Biology

Ileart & Lungs tffiffiffi

Cardiac Output :iiiiiijr;iilliijiiiti::

Every time your heart beats, a certain volume of blood is pushed out into the circulatory system. The cardiac output is that amount of blood which is pumped per minute by each of the two individual ventricles of the heart. We can define the cardiac output (for either ventricle in liters/minute) as being equal to the heart rate (in beats/minute) times the stroke volume (in liters/beat). This is shown in equation (2-1). The stroke volume is simply the amount of blood ejected by each ventricle during one beat of the heart.

If the average heart rate is T2beats per minute and the average stroke volume is 70 milliliters (or 0.07liters) per beat, then the cardiac ouput would be about 5 liters per minute. This value is for the average resting adult male. As we will see later, the volume of the cardiac output can be influenced by the diameter of the blood vessels in the periphery, the amount of blood returned to the heart by the superior and inferior vena cava (i.e., return of the venous blood), and the heart rate and force of ventricular contraction.

Cardiac Output = (Stroke Volume) (Heart Rate)

Copyright

@

by The Berkeley Review

90

(2-L)

The Berkeley Review Specializing in MCAT Preparation

Biology

Ileart & Lungs

Poiseuille's Law

Poiseuille's Law Not all of the arteries and veins in the circulatory system have the same diameter. flow of biood in blood vessels, established a relationship between the radius and length of a tube, the change in pressure between the two ends of the tube, the viscosity, and the flow rate of a fluid in that tube. This relationship, known as Poiseuille's law, is given in equation (2-2). Jean Poiseuille, a French physician studying the

Flow = AP ftRr = (P, - P, 'SnL ) 7IR*

SnL

(2-2)

in this equation

(2-2), AP is the pressure drop between the two ends of the tube (i.e., P1 - Pz), R is the radius of the tube, eta (r1) is the coefficient of viscosity, L is the length of the tube, and n/8 is a proportionality constant adjusting for the cross-sectional area of the tube. What can we say about this equation?

(a)

Notice that the fiow rate is proportional to R4. This tells us that the rate of blood flow is extremely dependent on the radius of the vessel. If the radius of the vessel were reduced by a factor of 2, then the rate of blood fiow would be reduced by a factor of 16. similarly, if the radius of the vessel were increased by a factor of '1,.5, then the flow rate would increase by a factor of 5.1.

(b)

The

(c)

The flow rate is also inversely proportional to the viscosity of the solution. This tells us that a high viscosity gives a 1ow flow rate.

(d)

The value of AP is provided for by the strength of the heart's contraction. In

flow rate is also inversely proportional to the length of the vessel. In other words, the longer the vessel, the slower the rate of flow. The shorter the vessel, the faster the rate of flow.

other words, the difference in the pressure is what drives ihe blood in the cardiovascular system.

Sopyright

@

by The Berkeley Review

The Berkeley Review Specializing in MCAT Preparation

Biology

Ileart

6e

Lungs

Diffusion

Diffusion is simply the process by which molecules randomly move from one place to the next. The molecular weight of a moiecule and the temperature of the medium have a lot to do with the velocity at which a molecule moves. Smaller

molecules tend to move faster than iarger molecuies. Similarly, a higher temperature provides more energy to a system and therefore imparts more thermal motion to the various moiecules in that system. you might think that because smaller molecules move relatively fast they would have no problem traversing their environment. However, don't forget that there are millions and millions of molecules within a given system and that each of those molecules, even thought they are moving at a given velocity, collide with their neighbors. These collisions tend to alter the path o{ the molecules, thus confining them to a random walk through their medium. Consider an imaginary sphere of water with a given radius. suppose we place some dye outside this sphere of water and ask how long it will take to reach the center. The answer clearly depends on the radius of that sphere. If the radius were 7 microns, it would take the dye about 5.4 seconds to reach the center. Flowever, if the radius were L centimeter, it would take about 11,000 seconds or a little more than 3 hours to reach the center. [As a reference a red blood cell is about 7 microns in length. There are about 1,400 red blood cells end to end in 1 centimeter.l what this is telling us is that simple diffusion is a rather poor way for a molecule to trek across long distances. The Law that governs diffusion is given in equation (2-3) where J is the net flux or net rate of diffusion (in moles per unit time, usually in seconds; that is, mol/sec), D is the proportionality constant called the diffusion coefficient, A is the area of the plane of interest (in cm2), and AC/Ax is the concentration gradient across that plane (in mol/cm4, because concentration is in units of mol/cri3 and distance is in units of cm). Since the net flux always proceeds down a concentration gradient, from a high concentration to a low concentration, we need to add a minus sign in front of this equation. [The minus sign indicates the direction of the flux or diffusion.] This equation is sometimes referred to as Fick's law (after the German physiologist who postulated it in the 19th century). In equation (2-3), what are the units of the diffusion coefficient, D?

r=-(DX^)f

Copyright

@

by The Berkeley Review

92

(DXAXCour - Cin) Ax

(2-3)

The Berkeley Keview Specializing in MCAT Preparation

Biology

fleart

8r Lungs

Osmosis

Osmosis In order to understand hydrostatic pressure we must first review osmosis. Recall that osmosis is simply the net movement of water from a region of high concentration to a region of 1ow concentration. General chemistry tells us that the concentration of pure water is 55.5 moles/liter. If we had a beaker of pure water and u,e added a solute to that beaker (say glucose), then we would decrease the concentration of the water in the beaker. Remember, when we add a solute nolecule (or molecules) we are occupying a volume of space that was once occupied by a water molecule. The more solute molecules we add to our beaker ,.f pure water, the more water molecules we will displace and the lower r.t'ill be re concentration of pure water. Jlucose, when added to a solution of pure water, does not ionize. It stays as :1ucose. Flowever, if we add a molecule of sodium chioride to a solution of pure .,-ater, it u'ill ionize into a Nae ion and a Cle ion. Because we now have added a .-.,lium chloride molecule which has dissociated into two ions in solution, we :.ar-e displaced (iowered the concentration of) the water molecules twice as much :-s rre would if we had added a glucose mo1ecu1e. \\hat this is telling us is that --:.e concentration of water in a given solution depends on the number of solute :a:ticles (e.g., glucose, Nae, or Cle) in that solution. We can define the total .-ir-rte concentration in our solution as the osmolarity, where one osmol is :-::prlv one mole of a molecule that does not ionize. If we had a 1M solution of :-r.ose, it would have a concentration of 1 osmol per liter. If we have a 1M .: -'.ihon of sodium chloride, we r,r,ould find that it wouid have a concentration of I osmols per liter (one from the Nae ion and one from the Cle ion). If we have a 1i0mM concentration of NaCl, then after ionization we would have 150 mM of :: rons and 150 mM of Cle ions or a total of 300 milliosmols (mosmols) per ,-:=:. Therefore, the osmolarity refers to the concentration of solute particles that : :.ave in our solution.

I

Ah

L

a

Hzo- >. oo lj

o

o

to toa a Oa a

'a )ao .o

Semipermeable Membrane

Permeable Membrane

F,gure 2-5 -

:

.: ::.ovement across a permeable

-::ose

rve have a U-tube

and semipermeable membrane.

with water and apermesble mernbrane as shown in

I fure 2-5a. Water is free to pass back and forth across this membrane and | : - irS€ of this the height of the water in each of the columns of the U-tube will r : --.: Sdrrl€. However, suppose we now replace the permeable membrane with a ""

''.'-'::'neable membrane and then add some protein to the right side of the U1s shown in Figure 2-5b. \44rat will happen? The level of water in the right

::

:,:::ht

O by The Berkeley Review

93

The Berkeley Review Specializing in MCAT Preparation

Biology

lleart & Lungs side of the tube drop. \Alhy?

Osmosis

will rise and the level of water in the left side of the tube will

In order to understand this we must examine the semipermeable membrane and the solutes which we have added to the right side of the tube in a little more detail. First, the concentration of solute on the right side of the tube is greater than the concentration of the solute on the left side of the tube. This established a solute concentration gradient in which the solutes on the right side of the U-tube want to diffuse down their concentration gradient to the solution on the left side of the U-tube. This cannot happen, because we have said that the membrane is semipermeable. In other words, the membrane will not allow these solutes to pass through only water. What about the concentration of the water? The concentration of the water on the left side of the U-tube is greater than the concentration of water on the right side of the U-tube. Again, a concentration gradient has been established that will allow water to diffuse from the left side of the U-tube to the right side of the U-tube. Since the membrane is permeable to water we find that water diffuses down its concentration gradient (from left to right). This leads to an increase in the volume in the right side of the U-tube and a decrease in the volume in the left side of the U-tube. The effect is to decrease the solute concentration in the right side of the U-tube. An equilibrium will eventualiy be established when the concentrations of both the water and the solute are equal on both sides of the semipermeable membrane. Once equilibrium is reached no more water will flow from the left side of the Utube to the right side of the U-tube. This is because the pressure has increased in the right side of the U-tube (because there is now a larger volume of solution pushing on the semipermeable membrane). The amount of pressure that stopped osmosis is referred to as the osmotic pressure (abbreviated as nosm). A direct measure of the osmotic pressure is the difference in the levels of water in the left and right sides of the U-tube. This difference, Ah, is referred to as the hydrostatic pressure (or fluid pressure) which can be abbreviated as PH2O. The osmotic pressure is proportional to the number of dissolved molecules in a solution and is represented in Figure 2-6. As we increase the concentration of the protein in solution (i.e., solute in solution) we find that the osmotic pressure increases as well. o

()O ov tr q

o [Protein]

Figure 2-6 Relationship between osmotic pressure and dissolved particles.

Now, let's return to the hydrostatic pressure generated by the heart which forces fluid out of the capillaries and into the interstitial space. Since the hydrostatic pressure in the capillaries turns out to be a bit greater than the osmotic "pulling pressure" of the soiutes in the blood, there is a net movement of fluid from the capillaries to the interstitial space and eventually into the lymphatic capillaries.

Copyright @ by The Berkeley Review

94

The Berkeley Review Specializing in MCAT Preparation

Biology

Ileart & Lungs

Lymphatic System

Lymphatic System The lymphatic system lies parallel to the systemic and pulmonary circulations. Tl'Lis can be seen in Figure 2-7.The lymphatic system ioliects the excess fiuid about 4 liters per day) that leaks into the interstitial space from the capillaries and returns it by way of the vena cava back to the ciriulatory system. Lymph :-.cdes located along the lymphatic system help to filter out foreign particles that :ould potentially lead to disease. If the lymph flow through the iymphatic system -"''ere biocked, edema wouid result. This is simply an increase in the inteistitial :-';id (because it cannot be reabsorbed by the 1ymphatic system). patients who :-ar-e heart surgery usually have swollen legs and ankles. This is because the --:art cannot pump the blood out fast enough, and as a result blood within the ---:art begins to back up. This translates to a back-up in the veins and eventually - lne lvmphatic system. Since gravity pools fiuid toward the lower extremities, :

*::fla results in the legs and

ankles.

Q

t-ymphatic capillaries

Pulmonary capillaries

Artery

.,:$

\ Vein

Heart

\ Systemic

-.nph

capillaries

:de

Q

Lymphatic capillaries

i.r1Ule 2-7

:

,

*::=:lC

SVStem.

..., 3

b1' The

Berkeley Review

95

The Berkeley Review Specializing in MCAT Preparation

Biology

Ileart & Lungs

Blood Ctotting

B[db i]iffiurfifig Blood clotting occurs via a cascade prlcess. Thrombin, which is involved in bibod clotting, is a serine protease. We can achieve amplification of a very weak signal by a cascade process. If we consider blood clotting, we will find that there is an intrinsic route (due to contact with some abnormal surface) and an extrinsic route (due to trauma to the tissue), both shown in Figure 2-8.

In the outline shown in Figure 2-8 we wili use the Roman numerals to represent the clotting factors. The subscript "a" means that we are dealing with the actiae

form of the molecule. Again, this would be some type of conversion of a proenzyme to an enzyme. Factor IX will be converted to Factor IXa by some intrinsic factor. Factor IXa wili be the trigger which will convert Factor X to Factor Xa. From the extrinsic portion of this scheme we start with some tissue factor which will convert Factor VII to the active form, Factor VIIa. Factor VIIa can also bring about the conversion of Factor X to Factor Xa. It is Factor Xa which is involved in the conversion of prothrombin (II) to thrombin (lla). Thrombin (IIa) wili convert fibrinogen (I) to fibrin (Ia). These fibrin fibers are then crossed-linked by Factor XIIIa (which is an enzyme called transglutaminase) to form the mature cross-linked fibrin ciot. Some of the serine proteases mentioned

in Figure 2-8 are Factors VII, VIIa, X+ process there are more than 15 different factors involved, and about 8 or so of them are serine proteases.

II, and IIa. In the blood clotting

Intrinsic Pathway

Factor

(Damaged Surface)

Extrinsic Pathway

o o r-------)

vil

IX

(Trauma) n

Factor IXo

Factor

I vrrr"

X t-----1

Factor VII

n

il

1'

Factor

VII^

Factor Xu

{}v^

Prothrombin (II)

Fibrinogen (I)

1,' !--------------- tsactorX

Thrombin (IIo)

vil

Fibrin (Io)

vfl *,,,. {transgturaminase} Crossed-linked Fibrin Clot

Figure 2-B The intrinsic and extrinsic pathways.

A lot is known about the biochemistry of blood clotting. In the conversion of Factor X to Factor Xa by Factor IXa, we find that we need an additional factor called Factor VIIIa. Individuals lacking Factor VIIIa have hemophilia (a sex-linked recessive characteristic). Factor VIIIa is sometimes called the antihemophiliac JLtLtUt.

Copyright @ by The Berkeley Review

96

The Berkeley Review Specializing in MCAT Preparation

Biology

fleart & Lungs HO

o il

Carboxylase enzyme

lil

l\

lt H -

requiring Vitamin K

L

and HCO3

CH.

l'

CHr I

C.

Oo//

\o

.

L__r'

0,

Blood Clotting

OH

ill C_N_C tt HC

o il H2

I

p

y '

Y-Crrboxyglutamate

o=c/

resrdue

I

^o

L-.-'/

Preprothrombin lragment

C -H

C:O

o^ I

*Can

l:,/

Prothrombin fragment chelated with calcium.

Figure 2.9 Cheiating action.

vitamin K is one of the fat-soluble vitamins that is found in green leafy

''-egetables. in addition, our intestinal flora can make a form of vitamrn K. \A{rat is the importance of this vitamin? Prothrombin exists in even an earlier form :ailed preprothrombin. In preprothrombin there are certain Glu residues which are carboxylated by a carboxylase enzyme. This carboxylase enzyme has an absolute requirement for vitamin K. The carboxylase enzyme adds another carboxyl group lo the gamma position of the first ten Glu residues located in the amino terminal region of preprothrombin. Thus, in the presence of vitamin K, l{CO3 and the carboxylase enzyme, we will be able to convert preprothrombin to

prothrombin. This structure now has a great ffinity (a good chelating agent) for lir.alent ions such as Ca2o. This is shown in Figure 2-9. (Calcium ions are essential for biood clotting.)

Platelet Membrane

following injury.

r", *Ca+ a

0C)

rl

"-\

,L-u C-H

*ca* o ooo u-L

rl \

C

a=a)H

I

CH.

NH:

t-

i/\

A section of prothrombin

Aftet cur is made. thlombin is released.

_A_

o Cut

Figure 2. I O Binding of prothrombin to

a membrane.

Prothrombin is next converted to thrombin in the presence of Factor Xa. The :onversion of prothrombin to thrombin in the presence of Factor Xa can be pictured as shown in Figure 2-10. Blood platelets have phospholipid molecules in their :rernbrane. The head of the phospholipid is negatively charged. This will allow Cop;,right O by The Berkeley Review

97

The Berkeley Keview Specializing in MCAT Preparation

Biology

Ileart & Lungs

Blood Clotting

the y-carboxyglutamate residues of prothrombin to bind via ca2e. Factor Xa also

has y-carboxyglutamate residues on it, and can also bind to the membrane via interaction with Ca2@. The enzyme Factor Xa wiil make cuts (at Arg-X residues) in the prothrombin molecule as shown. The portion of the prothrombin molecule that is cut away is called thrombin. Thrombin will drift away and convert fibrinogen into fibrin in the vicinity of the damaged area. It is fibrin that wili

{orm the blood clot. There is also an auxiliary factor, Factor Va, which is

involved in this process. So, what you need for this clotting process to take place is the (a) platelet membrane, (b) enzyme, (c) Ca2e ions, (d) an auxiliary factor, and (e) the substrate prothrombin. Thrombin can now act as a proteoiytic enzytne that converts fibrinogen to fibrin. Fibrinogen is a large solubie protein. Its solubility is due to an excess of negatively charged amino acids (Glu, Asp, Tyr-so4), particuiarly in the central domain of the molecule. The net charge in the central domain of the fibrinogen molecule is -8, while the net charge at the terminal ends is -4. If very large molecules have a net charge of zero, they will tend to come together. However, if we have an excess of negative charges or positive charges, the molecules will repel one another.

Fibrinogen

Fibrinopeptides

hFibrin

fl ll t,

Aggregation of

ribrin monomers

(

Fibrin Clot

o

Figure 2-l I Excess charge aliows for aggregation.

Thus, one way to make things soluble is to have an excess of charge. In order to convert a soluble protein to an insoluble protein one must remove that portion of the molecule contributing all those negative (or positive) charges. When fibrinogen is converted to fibrin we find that 4 Arg-Gly bonds are broken in the central domain. This releases four peptides containing an excess of negative charge called fibr

Copyright

@

by The Berkeley Review

inop

ep

tid e s.

9a

The Berkeley Review Specializing in MCAT Preparation

Biotogy

fleart & Lungs

Blood Clotting

Once these fibrinopeptides are released, the overall net charge in the central domain now becomes +5. The fibrin nLonlmer that is now forir.ed (due to the release of the fibrinopeptides) has the ability to interact with other fibrin nonomers through electrostatic interactions between the terminai and central domain regions of the polypeptide. This is shown in Figure 2-r2. This aggregation of fibrin monomers leads to the formation of the fibrin clot.

lhe clot that is initialiy formed is called a soft clot. The hard clot involves a :-rrther step, in which there is a cross-linking via the enzyme transglutaminase -r'r Factor XIIIa). Cross-links are formed between the individual subunits that :--ake up the aggregate. Those cross-iinks occur between Gln and Lys resid.ues as !.-o\\'n in Figure 2-12. This type of reaction is referred to as a trnnsaminntion - . :lian. ";

Fibrin-

CHz

-

CHz

C- NH2

-

NH3-

GIn

I

il XIIIa U

Fibrin-

cHz

-

CHz

-

(CHr4-Fibrin

Lys (transglutaminase)

C- N- (CHz)+-Fibrin

Cross-linked

Fibrin Clot F1Eure --tng Ieads to a hard clot.

damaged area has been repaired, a serine protease called plasmin in the fibrin ciot in order to dissolve it into smaller ::'1: rragments (to remove the clot). Tissue plasminogen actipstor (TpA) ;;-,. plasminogen into this active protease. ;'Le

'.'zes specific regions

:-;

:::rirn to vitamin K for a moment. vitamin K is one of the fat-soluble ,:=,-:.s 'hat is found in green leafy vegetables. in addition, out intestinal flora - :::-

6CO2 + 6I{2O + Energy

(2-47

In

:

general, respiration is the process by which oxygen is brought to the cells of the tissues and carbon dioxide is removed ur u -urt" product. As we breathe air into our lungs it first enters our system by way of the nose or mouth. The air passes form the oral cavity to the pharynx, into the larynx, and then down the trachea. At the end of the trachea the air passes into two tubuiar passageways called bronchi. One bronchus enters into eich lung and continues to divide into smaller passageways called bronchioles, ending eventually in the functional units of the lungs which are the alveoli. Each alveolus consists of a single layer of epithelial cells juxtaposed to a very thin basement membrane. Sulrou1,air1g each single alveolus is a capillary network. The epithelial cell layer of eacfi alveolus and the endothelial layer of the capillaries are separated from each other by a very narrow interstitial space (if they are separated by an interstitial space at all). This means that the air in each alveolus and the blood in the capillaries are separated by a very small distance (about 0.2 to 0.3 pm, compared to the average diameter of an erythrocyte, which is about 7 pm). Th" t*o irrrg, are composed of millions of alveoli, and if they were all laid flat on a surfac"e, their combined total surface area would be between 70 m2 to 100 -2 1*ni.h i, about the size of a tennis court). Therefore, Iarge quantities of oxygen in the alveoli of the lungs can quickly be equiiibrated wlth-the blood in the "capiliaries bec.auss of the large surface area and the thin barrier to diffusion for gas exchange.

The air that we breathe on a normal day is composed of roughly 7g"/o N2,21"/. oz' 03% Co2, and 0.7"/o H2o. [These values vary slight from textbook to textbook.] Because al1 o{ these gas molecules are (normally) quite far apart, they tend not to interfere with one another. This means that the pressure exerted bv one gas rs independeruf of the pressure exerted by ail of the other gases. hr oth& words, the sum of each of these individual gas pressures equals the total pressure of this mixture of gas. The partial pressure of a gas (e.g., pN2, po2, pco2, etc.) is therefore a measure of the concentration of a given gas (such as 02) in a mixture of gases (i.e., the air). If we add up all the partial pressures of the individual gases in the atmosphere, we will come up with the atmospheric pressure. At sea level the atmospheric pressure is 760 mmHg. partial pressure of oxygen gas (Po2) at sea level? Roughiy 21o/, of the total atmospheric pressure at sea levei is oxygen gas. Therefore, the partial

\MFrat is the

pressure of oxygen gas at sea level is about 160 mmHg (from 0.21 x 7G0 mmHg = 16o mmHg). what happens to the parti.al pressure of oxygen as we ascend to the Copyright

@

by The Berkeley Review

lol

The Berkeley Review Specializing in MCAT preparation

Biology

Ileart & Lungs

Gases

top of a high mountain? The concentration of oxygen in the air on this mountain top will still be about 21"/o.However, as we increaJe our altitude the atmospheric pressure begins to decrease. The result is a decrease in the partial pressure of oxygen.

How does a gas behave in a iiquid? Consider an open glass of water sitting on a table as shown in Figure 2-L4. The surface of that water is constantly being bombarded with gas molecules from the air.

'.,., "'l ,',,

',9j,,

............

Watei,,

Figure 2-14 Gas movement across a water surface.

when a gas molecure such as 02 comes in contact with that water surface it can dissolve in the liquid. The number of O2molecules which dissolve in the water is directly proportional to the partiar pressure of the 02 gas. Just as 02 molecules in the air can hit the surface of the water and dissolv;; th" ltq*d so too can 02

molecules

in the liquid hit the surface of the water and

into the air.

Th.erefore, at equilibrium the number of 02 molecules "slcape dissolving the water will equal the number of 02 molecures leaving the water. In otherinwords, the partial pressure of oxygen in the gas phase (air) Is equar to the partial pressure of oxygen in the liquid phase (water), as shown in equation (2_5).

(Por)gu. = (Poz)liquia

(2-s)

W5 cl say that the pressure of the air acting on the membranes of the epithelial cells in the alveoli of the lungs is the sum puitial pressures of all the gases in the air. Thus, the total pressure for any given gas molecule is directly proportional to the concentration of that gas molecure in the air. For example, -" .u1 account for

the many factors that a{fect the rate of diffusion of a gas into a liquid by considering equation (2-3). In this case J is the flux or difiusion rate of ihe gas molecule across the membrane, D is the diffusion coefficient, A is the area of the plane the molecule is diffusing across, C is the concentration of the moiecule, and x is the distance of diffusion. Since the concentration of a gas is proportional to the pressure of the gas, we can write that as shown in equati"on (2-6).

J_ Copyright

@

by The Berkeley Review

(DXA)(C1O, - C2o')

Ax t02

s

-

(DXA)elo, - p2o) Ax

(2-6)

The Berkeley Review Specializing in MCAT preparation

Biology

tleart

6r Lungs

Gases

The respiratory passages have a unique anatomical structure. Let's consider a :ross-section of one of these passageways and work our way from the lumen rutward. The epithelial cells which line the lumen of the passageways to the end ri the bronchioles have cilia which are continualty beating towards the pharynx. Scattered among these epithelial cells are glands which secrete mucus. As the :ilia beat towards the pharynx the mucus is moved upward towards the oral :ar"ity. Any foreign matter that has become trapped in the mucus is eventually

::ansported to the oral cavity where it is swallowed in a normal reflex action. ,.-\'re of the reasons that smoking is bad for you (besides its carcinogenic nature) is ::.at it decreases the activity of the cilia and lowers the body's defenses agarnst -'-::rg infection by bacteria that can enter the respiratory tract on airborne dust ::rticles. Immediately beneath the layer of mucus is a iayer of fiuid in which the :rlia operate. Air that flows within the passageways of the respiratory tract is

'..-arrned and moistened.

-:.: upper

passageways of the respiratory tract maintain their opening by means - cartilage rings that surround most of the diameter of the passageways. By the ,-:-Le the bronchioles are reached the cartilage has disappeared. smooth muscle is -l-,ji-rd in almost all areas of the respiratory tract where there is no cartilage. For = '.anple, the walls of the bronchioles are mainly smooth muscle. The bronchioles : :re lungs are innervated by nerve fibers from parasympathetic nerves (which ::-,'ei ir-t the vagus nerve). -

{sthma is usually caused by an allergic hypersensitivity to airborne antigens ---ch have entered the respiratory tract. The direct result is to cause the mast -=---' rr-ithin the bronchioles to release a number of different substances which -,'--'e the smooth muscle surrounding bronchioles to spasm and constrict.

ltechanics of Breathing -' = thoracic cage which contains the lungs is separated from the abdomen by a :,-::i oi skeletal muscle and connective tissue called the diaphragm (see Figure --i5. The lungs themselves are encased in a pleural membrane. The visceral :-eura covers the lungs, while the parietal pleura adheres to the diaphragm and : = - l-.est wall. Between the viscerai pieurai and parietal pleura is the intrapleural .:ace rr'hich contains a watery fluid. As the muscles of the diaphragm contract -: .- rull the diaphragm itself downward. Simultaneousiy the muscles of the rib .:= ccntract and cause the rib cage to move upward and outward. Both of these -

"-::-s enlarge the area of the thoracic cage that contain the lungs.

:

-= the diaphragm is attached to the parietal pleura this pleura is also pulled ard. The watery fluid in the intrapleural space is rather indistensible and ,-= : -:Iling of the parietal pleura translates through the intrapleural fluid. It also --: 'rre visceral pleura downward as well. Enlargement of the thoracic cage .

,

",-:,'i!

:::.ls the lungs and creates a subatmospheric pressure in the alveoli. Air -,.:.=. dorvn its pressure gradient from 760 mmHg at sea level to whatever lower r::.i jl€ i.s found in the lungs at inspiration.

: :- u1e muscies of inspiration stop contracting, the elastic tissue found within '= --:.lracic cage and lungs returns to its normal length. Air within the alveoli is ":::esseci and forced out through the passageways of the respiratory tract -:-:.i expiration.

-:it

O bv The Berkelev Review

r03

The Berkeley Keview Specializing in MCAT Preparation

Biology

tleart & Lungs If the lungs were to

Gases

become separated from the visceral pieura, they would

collapse. This is because they have no anatomical structures io maintain rigidity. one way to separate the rungs from the visceral preura is to receive a very ltrorrg blow to the chest area (as often happens during football games). To the pharynx and oral cavity

f

Larynx

E) Intrapleural

Cartilage r

space

nngs

Parietal

pleural Diaphragm

Figure 2'15 The thoracic cage.

Copyright O by The Berkeley Review

to4

The Berkeley Review Specializing in MCAT preparation

Biology

Ileart & Lungs

Gas Exchange

Gas Exchange It is important to remember that

molecules u'il1 diffuse from a site of high

concentration to a site of low concentration. As deoxygenated blood (a dark red color) is coming back from the tissues it enters the right atrium via the superior and inferior vena cava and then passes into the right ventricle, where it is pumped out to the lungs. The PO2 is about 40 mmHg and the PCO2 is about 46 mmHg as this deoxygenated blood enters the capillaries surrounding the alveoli of the lungs. The PO2 and PCO2 within the alveoli are about 105 mrnHg and 40 mmHg, respectiveiy. Passage of the blood through the capillaries is relatively slow and an equilibration can be reached between the gas exchange in alveoli and the capillaries. Oxygen will diffuse down its concentration gradient from the alveoli to the capillaries, and carbon dioxide will diffuse down its concentration gradient from the capillaries to the alveoli. As the oxygenated blood (a bright red color) Ieaves the capillaries of the lungs and enters the left atrium of the heart, the PO2 is about 100 mmHg and the PCO2 is about 40 mmHg. See Figure 2-16. PO2

- 160

PCO2 = 0.3

PO2,= QQ P,COz= 46

PO2 = 100 PCOz = 40 Capiliaries at level of alveoli

I H

Capillaries at level of tissues

PO2=40,-

PCOr'='46

FO2=l[g

i

: FCO2=49

PO2 < 40

Figure 2- 16 Gas exchange at the level

ofthe lungs and tissues.

Oxygenated blood will pass from the left atrium to the left ventricle of the heart and then be pumped to the tissue capillaries. At the level of the tissues the PO2 in the cells (depending on which cells you are considering) is less than 40 mmHg, while the PCO2 in those cells is greater than 46 mmHg. Oxygen will diffuse from the blood in the capillaries to the ceils while carbon dioxide wiil diffuse from the cells to the blood in the capillaries. The deoxygenated blood that leaves the tissue capillaries returns to the right atrium of the heart via the venous system. Copyright O by The Berkeley Review

l05

The Berkeley Review Specializing in MCAT Preparation

Biology

Ileart & Lungs

Gas Dxchange

Oxygen can travei in the blood by being dissolved in the blood itself or by being

bound to a transport protein in the red blood cells (erythrocytes) called hemoglobin (abbreviated as Hb). Since oxygen is rather insoluble in water, not much is actuaily dissolved in the blood and transported in that manner. However, since hemoglobin has such a high affinity for oxygen, more than 987o of the oxygen in contact with hemoglobin is transported by this protein.

Hemoglobin is a protein that is composed of four polypeptide subunits. When these subunits interact with each other to form the hemoglobin molecule, they give hemoglobin a quaternary structure. Located within each of the four polypeptide subunits is a heme prosthetic group that has an iron atom in the center which is in the ferrous (Fe2e, oxidation state. Since one hemoglobin molecule has four binding sites for oxygen, there is a potential for 4 02 moiecules to bind to one hemoglobin molecule. Every time hemoglobin takes up 02 from the blood more 02 can leave the gas phase in the alveoli and enter into the liquid phase in the blood. This increases oxygen's solubility in the blood.

Oxygen Saturation Curve for tlemoglobin In Figure 2-17 we have a plot of the percent saturation of hemoglobin with oxygen as a function of the partial pressure of oxygen at various places in the body. Recall that when the PO2 is about 100 mmHg, we are at the ievel of the alveoli and when the PO2 is about 40 mmHg, we are at the level of the tissues. [It is best to read this curve from right to left and from top to bottom.] Venous

Blood

pH7.4

Arterial Blood 02 released

to tissues

€80 b0

Extra 02 released

f;oo

to tissues

qi

'E

Shift due to:

40

Decrease in pH Increase in temperature Increase in 2,3-DPG

! d a

*o

o (.)

?O

Resting Muscle

Working Muscle

Figure 2- l7 Oxygen-hemogiobin dissociation curve

At the level of the capillaries (PO2 = 100 mmHg) surrounding the alveoli roughly 98'/" of the hemoglobin is saturated with oxygen. As the blood passes to the tissues (PO2 = 40 mmHg) roughly 25"/' of the oxygen that was bound to hemoglobin is given up to the tissues. This can be seen in Figure 2-17. Copyright

@

by The Berkeiey Review

l()6

The Berkeley Review Specializing in MCAT Preparation

Biology

Ileart & Lungs

Gas Exchange

Effects of ptl, Temperature, and 2,3,BP(i The upper oxygen dissociation curve shown in Figure 2-17 is for normal conditions in which the body temperature is about 37 "C and the blood pH is about 7.4. If we were to decrease the pH of the blood (to a pH of 7.2) or increase the temperature, then the oxygen dissociation curve would shift to the right and downward as shown in the lower curve in Figure 2-17. This happens when you exercise. The same iype of shift occurs when a by product of glycolysis, 2,3bisphosphoglycerate (abbreviated as 2,3-BPG), binds to hemogiobin. 2,3-BPG is usually slnthesized in increased amounts when the body has been deprived of oxygen for an extended period of time (e.g., when you visit high altitudes).

AII three of these interactions with hemogiobin (the lowering of the pH, the increase in temperature, and the binding of 2,3-BPG) cause hemoglobin to release more oxygen to the tissues at the same partial pressure of oxygen as in the standard case. The oxygen dissociation curve can be shifted to the le{t and upward by reversjng these interactions.

Carbon Dioxide How is CO2 carried in the blood? It can be carried by (a) dissolving in the plasma and the red blood ceiis, (b) binding to a specific site on the hemoglobin molecule, or (c) in the form of bicarbonate ions (HCO3e), About 70% of the carbon dioxide rs carried in the blood in the form of bicarbonate ions, roughly 20o/o is carried by the hemoglobin itself, and about 10"/' is dissolved in the plasma and red blood cells. See Figure 2-18.

7jVo

Plasma

Red Blood Cell

CO2,

+ H2O

anhydrase

-:-+

20Vo

H2CO3

Hb-COo,

/

^

^

H@ + HCO3e 57a CO,t

57a COo dissolved

Capillary

CO" from

tissues

Figure 2-IB ,las exchange at the level of the tissues.

the level of the tissues the PCO2 is greater than 46 mmHg. However, in the :Iood of the capillaries it is about 40 mmHg. Therefore, CO2 will diffuse down its :oncentration gradient and into the blood. Some of the carbon dioxide will jissolve in the biood plasma, some will dissolve in the red blood cell, and some ,,, il1 bind to hemoglobin. The remaining CO2 will react with water and be :onverted to carbonic acid (H2CO3) by the enzyme carbonic anhydrase. Carbonic acid will ionize to the bicarbonate ion and a proton (H@). Bicarbonate will diffuse rto the biood plasma and be carried by the circulatory system to the capillaries

-\t

-,: the lungs. See Figure 2-18.

-opyright

@

by The Berkeley Review

t07

The Berkeley Review Specializing in MCAT Preparation

Biology

tleart

E(

Lungs

Gas Exchange

At the level of the capillaries in the lungs, the PCO2 in the blood is about 46 mmHg, while the qCOZ in the alveoli of the iungs is about 40 mmHg. Once again, CO2 wiil diffuse down its concentration gradient and into the alveolar space. The CO2 that is dissolved in the plasma and in the red blood cell diffuses into the alveoli as does the CO2 that was bound to the hemoglobin. Bicarbonate ion in the plasma will diffuse into the red blood cell and combine with a proton to become carbonic acid. Carbonic anhydrase will convert carbonic acid to water and carbon dioxide. The CO2 diffuses into the lungs. This is shown in Figure 219.

Red Blood Cell

CO2

+ HzO

H2CO3

'

Acquired immunity may be achieved through natural or artificial means. For instance, a person may be exposed

B.

and

secondary response to an antigen. IgG, a monomer,

Acquired immunity is the resistance to disease that an organism develops during its lifetime. Immunity may be

A.

Passage II

of

microorganisms that caused cowpox as

a

vaccination against the microorganisms of smallpox. What type of immunity was conferred in this casl?

295

A. B.

Artificially acquired active immunity. Artificially acquired passive immunity.

C. D.

Naturally acquired active immunity. Naturally acquired passive immunity.

The Berkeley Review Specializing in MCAT preparation

a

Biology 13.

Acquired Immunity

Passage

Blood serum is subjected to electrophoresis in order to separate the proteins. Antibodies are present in the gamma globulin fraction. The following diagram shows serum proteins following gel electrophoresis:

Direction of migration

Cathode

Anode

(-)

(+)

B Globulins

Albumin

In this diagram, which protein is the largest,

based

on its migration pattern?

A. B. C. D.

albumin

a-globulin B-globulin y-globulin

14. If

a newborn is orphaned at birth, which process would provide more antibodies for the child?

I.

Feed the chiid breastmilk from another nursing

mother.

II. Seclude the child in a sterile hospital unit. IfI. Vaccinate the child immediatelv to

all

childhood diseases.

A. B. C.

D.

Copyright

i only I and II only II and III only I, II, and III

@

by The Berkeley Review

The Berkeley Reviev Specializing in MCAT Preparation

Biology Passage

III

Major llistocompatibility Complex (MHC)

(Questions 15-21)

Passage III

Endosomes are membrane vesicles that often contain surface proteins and their associated ligands. Endosomes are known to contain proteases, which are beiieved to degrade the invariant chain and allow the molecule to bind peptide. However, a small part of the invariant chain (known as CLIP) remains at the binding cleft until a DM

Transplantation antigen proteins expressed on the of a cell and recognized by the immune system are encoded by the major histocompatibility complex (MHC) genes. The MHC genes are of two fundamental types, class I and class II. While mouse and human populations carry more than one hundred forms of the molecules, only between three and six of each class are surface

molecule (an MHC class

Il-like molecule)

enters the

endosome and binds CLIP, actively removing it from the MHC class II molecule. At this time, the MHC class II

molecule, bound with peptide,

expressed.

is placed on the cell

surface-

Both classes of MHC molecules are involved with antigen processing, which includes the ingestion of antigens, the fragmentation of antigens into peptides, and

15.

the binding of these peptides to MHC molecules. The formation of the MHC-peptide complex is a critical event in the effective elimination of intracellular parasites. The peptides associated with MHC class I have invariably

The amino acid sequence of both the MHC class I and MHC class II molecules should show:

A. B. C. D.

been found to originate from a cell's own proteins, while the peptides found bound to MHC class II are normally located on the outer membrane.

The MHC class I protein usually binds peptides that are eight to nine amino acids long. The two ends of the peptide chain bind to discrete binding sites located in the cleft of the MHC class I molecule (Figure 1). The binding cleft of an MHC class II molecule is similar in shape to that of an MHC class I molecule, but the MHC class II molecule usually binds peptides in the middle of the cleft (Figure 2).

hydrophilicregions. hydrophobicregions. both hydrophobic and hydroapathetic regions. both hydrophilic and hydrophobic regions.

16. On the surface of a cell infected by majority of MHC class I molecules

a virus,

the

have bound

peptides that originate from:

A. the virus. B. akillerTcell. C. a B cell. D. a host cell.

MHC ciass

II

17.

molecule

A. B. C. D.

Membrane

,\

MHC class II molecules have been shown to assemble in the endoplasmic reticulum. However, immediately after their synthesis, the MHC ciass II subunits associate with a

A. B. C. D.

class II molecules and causes the MHC class II molecules to leave the Golgi complex and fuse with endosomes.

Berkeley Review

are formed by which

of the following

processes?

invariant chain inhibits peptides from binding to the MHC

@ by The

Shine-Dalgarnosequence. signal peptide sequence. pyrimidine-richsequence. purine-richsequence.

18. Endosomes

third molecule known as the invariant chain. The

Copyright

The two protein subunits that constitute the MHC class II molecule are MOST likely to contain a:

297

Megacytosis Exocytosis Endocytosis Transcytosis

The Berkeley Review Specializing in MCAT Preparation

Biology

Major llistocompatibitity Complex (MflC)

19. The protein DM is structurally

A. B. C. D.

20.

Passage III

similar to:

the invariant chain.

MHC class lI. CLIP.

MHC class I.

Which of the following statements is true regarding peptides bound to MHC class I and MHC class II proteins?

I. Peptides bound to MHC class I should exhibit

II.

2I.

a greater size variation. Peptides bound to MHC class a greater size variation.

II should exhibit

III.

The amino acids binding to a cleft should be conserved despite the variety of peptides the cleft can bind.

A. B. C. D.

I only II only I and III only II and III only

Which of the following is LEAST likely to be a defense mechanism used by a pathogen to deter the antigen processing system?

A. B.

Suppression

of MHC molecules early in

an

infection

Production of molecules that bind to MHC I molecules in the ER and prevent cell

class

C. D.

Copyright

surface expression

Production

of a

transcription factor that of the MHC

increases the transcription rate gene early in an infection Suppression of the DM molecule

@ by

The Berkeley Review

294

The Berkeley Review Specializing in MCAT Preparation

Biology

IgA Antibody Experiment

IV (Questions 22-27)

Passage

The graphs in Figure 2 represent the numb,er o: trg,\ secreting cells in mouse lung tissue I and I rl eek" following exposure to a virus carrier alone or a r-irus canier plus the IL-6 gene.

In the following experiment, researchers studied the role of the cytokine, interleukin-6, as a factor in the response of immunoglobulin A (IgA) to foreign molecules in mice. IgA is the antibody group that is released from epithelial surfaces in secretions such as saliva or breast milk. IgA often represents a first line of defense against

22.

invading pathogens.

L Mouth

II.

The IgA responses of mice unable to make IL-6 (IL-6) and control mice (IL-6+) were studied. Ovalbumin, an egg

Immunoglobulin response is depicted in Figure q)

40 30

a_)

20

O

A.

1.

50

C)

10 (-)

0

B.

I only I and II only

C.

II

D.

I, II, and

IgM

IgA

IgG

Figure

IgE

A.

1

Experiment 2

I

IL-6- and IL-6+ mice were immunized with a virus construct carrying the mouse IL-6 gene. The virus was theorized to insert the IL-6 gene into the DNA of the host

C.

D.

cells it infected.

3

and

23. Which of Figure

Lr

Small intestine

IU. Urethra

protein, was added to the mouse intestinal mucosa.

o

Which of the following tissues are lined by epithelial tissue?

I

Experiment

namgn m

III only

III

the following statements is TRUE of

1?

IL-6- mice produce higher ievels of IgA than IL-6+ mice. IL-6- mice produce lower levels of IgA than IL-6+ mice. The presence of IL-6 did not affect the production of IgA. IL-6- mice respond strongly to ovalbumin.

100

Q9 bo ri -x

IL.6+ carrier alone

Q(

With virus

0!

LA \C)

IgA is composed of what type of molecules?

With virus carrier plus IL-6 gene

A. B. C. D.

;1 O

9n

t2l2

Amino acids Fatty acids Sphingomyelins Phospholipids

Weeks following innoculation of mice

3o-

With virus carier plus IL-6 gene

1oo

Which statement is TRUE of Figure 2?

Ug' :9q

A. The virus carrier

OO (.) e

v)s

B.

;aO

.tfF!

Qh

€o

alone transformed both

strains of mice. Restoring IL-6 in the IL-6- mice improved IgA

production.

1212

C.

IL-6- mice were hypersensitive to the virus

D.

IL-6+ mice did not respond to the virus carier.

carrier.

Weeks following innoculation of mice

Figure 2 ti

Copyright O by The Berkeley Review iti

&

299

The Berkeley Review Specializing in MCAT Preparation

Biology 26.

IgA Antibody Experiment

Passage IV

The researchers did not report IL-6 concentrations in the blood. Why is this the case?

A. B. C. D.

Cytokines are hormones and act while passing through the entire circulatory system. Hormone concentrations cannot be measured in blood. IL-6 never leaves the cell that produces it. Cytokines are local hormones and often act without passing through the entire circulatory system.

27.

What term refers to the medical alteration of genes to correct an inherited or acquired disease?

A. B. C. D.

Copyright

Gene therapy

Vaccination Geneticimmunization Pleiotropy

@

by The Berkeley Review

500

The Berkeley Kevier Specializing in MCAT Preparatiot

Biology Passage

Complement System

V (Questions 28-33)

Complement was given

29.

its name because it

D. 30.

A. B. C. D.

Complement consists of about 20 interacting proteins. The components involved in reactions are known as C1C9, factor B, and factor D. The rest of the proteins are involved in the regulation of this system. These proteins circulate in the blood in an inactive form unless aitivated directly by an invading microorganism or indirectly by an immune response. The final result of activation is assemblage of the late complement components (C5-C9) into a membrane attack complex.

31.

lymph nodes. bone marrow. thymus. spleen.

The classical pathway is usually activated

A. B.

polysaccharides on a microbial envelope. There are two distinct pathways of eariy component activation. Cl, C2, and C4 belong to the classical pathway and is triggered by antibody binding. Factor B and D belong to rhe alternative pathway and are triggered by microbial polysaccharides. Both pathways wiil act on C3, a central component in the complement system.

C. D.

32.

by IgG or

antigenic determinant. variable region of the antibody. cell membrane. constant region of the antibody.

As stated in the passage, protease cleavage acts to expose

a membrane binding site on the

larger

fragment. The most likely reason for this is to:

4. B.

All early components

and C3 are proenzymes that are activated by each other through proteolytic cleavage. As each proenzyme is cleaved, it is activated to generate a serine protease which cleaves the next proenzyme in the sequence. Each activated enzyme cleaves many molecules of the next proenzyme in the chain. The cleavage normally exposes a membrane binding site on the larger fi'agment and liberates a small peptide fragment into the blood stream. The C3 molecule is eventualiy cleaved, with its larger fragment binding both the cell membrane and C5. Activation of C5 will initiate the spontaneous assembly of C5 through C9, forming the membrane attack complex.

avoid precipitation of complement proteins.

C.

have the larger fragment act as a diffusible signal in the bloodstream. confine complement activation to the cell

D.

inhibit the next reaction in the

surface where it began.

cascade

sequence.

33.

During the proteolytic complement cascade, ser,eral small biologically active fragments are generateci. One of the end results of these molecules' activir;, is an increase the permeability of local blood vessels. Which is the most likely explanation for such an increase?

A.

passage, complement may

normally help:

B.

produceantigen-antibodycomplexes. destroyantigen-antibodycompiexes.

C.

solubilizeantigen-antibodycomplexes.

D.

precipitateantigen-antibodycomplexes.

by The Berkeley Review

a

to the:

The early complement components are activated by

@

the

igM antibodies bound to antigens on the surface of a microorganism. The Cl complex mosr likely binds

either antibodies bound to a microorganism or by

Copyright

The production of antibodies used in

complement process would be greatly affected b;' disease of the:

skin, joints, and brain causing the destruction of tissue.

A. B. C. D.

focuses the complement system away from cel1 membranes. does not take place in the alternate pathway. provides a means of amplification, ultimatelv leading to many MACs. uses serine proteases at all serine residues in a protein.

B. C.

and attracting phagocytic cel1s to the site of infection. Individuals who are complement deficient also suffer from immune complex diseases, in which antibodyantigen complexes precipitate in small blood vessels in

28. According to the

The proteolytic cascade described in the passage:

A.

complements the action of antibodies and is the principal means by which antibodies defend vertebrates against most bacterial infections. A system of serum proteins are activated to form a membrane attack complex (MAC) which forms holes in microorganisms. Complement also amplifies the defense system by dilating the blood vessels

Passage V

501

These molecules stimulate the histamine from T lymphocytes. These molecules stimulate the histamine from basophils. These molecules stimulate the histamine from macrophage. These molecules stimulate the histamine from erythrocytes.

secretron of

secretion of secretion of secretion of

The Berkeley Review Specializing in MCAT Preparation

Biology

Myasthenia Gravis/Autoimmune Diseases

VI (Questions 34-40)

Passage

35.

Myasthenia gravis is a rare, chronic, neuromuscular It is characterized by skeletal muscle weakness and fatigability in response to repeated contraction. Resting partially restores muscle stre;gth. The muscies of the eyes, face, jaw, and neck are usually affected first. As

Passage VI

Shown below is a diagram of a neuromuscular junction. Which number indicates an acetylcholine receptor?

disease.

the disease progresses, weakness spreads to rhe fn severe cases, all the

extremities and the diaphragm. muscies are weakened.

Research indicates myasthenia gravis

is

an

autoimmune disorder in which antibodies are produced to the acetylcholine receptor that is present at the neuromuscular junction. Antibodies to the acetylcholine receptor have been found in 85Vo of patients with

generalized myasthenia. Although the mlchanism for antibody production is unclear, one hypothesis is that certain thymus cells that resemble musci" (myoid cells) are.damaged by a virus. The virus may have a molecular region that mimics part of the acetylcholine receptor, such as-the herpes simplex virus. A virus may damage myoid

cells so that antibodies are produced against

tt"-

By whatever mechanism, the viraf infection

antibody production.

a a

I ol

air"ttty. induces

The actual interaction between the antibody and the receptor is not fully understood. The antibody may block the receptor, it may cause faster receptor breakdown, or it may promote complement-mediated damage.

A. I B.II c.m D.IV

Autoimmune diseases as a whole are relativelv In the following table is a list of soml autoimmune diseases and the antibodies produced in the common.

36.

Which of the following is NOT an example of autoimmune disease?

disease state:

A.

Antibody Against

Disease Type I

Destroys B cells

Graves' disease

Thyroid stimuiating

Stimulares TSH

hormone receptor

receptor on thyroid

Multiple

Myelin basic protein

Disrupts

sclerosis

(hypothesized)

myelination

Glomerulonephritis

34,

Effects

B celis of pancreas

diabetes

Basement membrane

B. C. D.

37.

Copyright

B.

Destroys a variable

C. D.

of glomular capillaries number of glomeruii

by The Berkeley Review

Which of the following is NOT a symprom of A.

38.

Increased metabolic rate. Weight loss.

Lethargy.

Hyperactivity.

Secretion

of which of the following from

the

pancreas is halted by antibodies to the B cells?

An inhibitor of acetylcholinesterase. An immunostimulant. A paralytic agent, like curare. An inhibitor of acetylcholine synthase. @

Type II diabetes. Addison's disease. Myasthenia gravis. Graves' disease.

Graves' disease?

Which drug could be given to counteract the effects of the antibody produced in myasthenia gravis?

A. B. C. D.

an

A. B. C. D. 302

Glucagon.

Insulin. Bicarbonate. Digestive enzymes.

The Berkeley Review Specializing in MCAT preparation

Biology

Myasthenia Gravis/Autoimmune Diseases

Passage VI

How can a viral infection lead to an autoimmune disease?

II.

III.

A. B. C.

D.

40.

The virus resembles a "self'molecule, leading to antibodies that cross-react with other body molecules The virus damages a cell so that unrecognized cell proteins are released, causing antibody production The virus resembles a "non-selfl' molecule, leading to antibodies that cross-react with other body molecules

I only I and II only II only II and III only

Often patients with autoimmune diseases are treated

with corticosteroids to reduce immune responses. Which organ in the body produces corticosteroids? A. B. C.

D.

Copyright

Pancreas. Adrenal gland. Pituitary gland. Thyroid gland.

@

by The Berkeley Review

303

The Berkeley Keview Specializing in MCAT Preparation

Biology Passage

Glucose, Glucagon, & Insulin

VfI (Questions 4l-46)

Passage Vtr

41. During times of stress

the importance of having

adequate levels of glucose, for utilization as energy, is dependent on hormonal secretions from the pancreas. Which choice below will BEST readv the

The regulation and metabolism of carbohvdrates in the

body are controlled by the pancreas. The endocrine component of the pancreas, the islets of Langerhans, are specifically responsible for carbohydrate control. These small clusters of cells imbedded within the exocrine portion of the pancreas contain peptides with specific

body for stressful situations?

A. B.

hormonal activity.

C.

Glucagon, secreted by the a cells (or A cells), liberates glucose from storage areas in the body, stimulates glucose production, increases lipid concentration in the blood stream by releasing free fatty acids, and increases the production of ketones. u cells are stimulated to secrete glucagon during increases in plasma amino acid levels, cortisol secretion, exercise, and sympathetic nervous system stimulation. Inhibition of glucagon is promoted by

D.

increases in plasma glucose, ketone, free fatty acids, insulin, and somatostatin levels.

Increased levels of glucagon and insulin.

Increased levels of glucagon and decreased levels of insulin. Decreased levels of glucagon and decreased levels of insulin. Decreased leveis Ievels of insulin.

42. Ketosis is

of glucagon and

increased

developed from an increase

in

the

conversion of free fatty acids to ketone bodies. These ketone bodies are an important source of energy in times of fasting. However, prolonged

Insulin is secreted by the B cells (or B cells) and functionally is important in increasing the storage of

ketosis will lead to a plasma acidosis due to liberated hydrogen ions from ketone bodies. A patient witb

glucose, fatty acids, and amino acids in the cells of target tissues. Furthermore, insulin decreases the release of glucose, mannose, amino acid, and glucagon plasma levels. Many intestinal hormones also stimulate insulin secretion. Parasympathetic stimulation of the cells will B increase insulin secretion, while sympathetic stimulation

acidosis

will develop

shortness

of

breath.

dehydration, hypervolemia, and hypotension. In severe cases the acidosis and dehydration will depress consciousness to the point of coma. Which choice below will lead to the development of ketosis

will inhibit secretion.

and acidosis?

Somatostatin is a peptide secreted by 5 cells (or D cells) in the islets. Somatostatin inhibits the production of both insulin and glucagon, and it acts as a regulator of

islet secretion.

The effects ofinsulin and glucagon target very specific

regions of the body where their cellular actions occur. Insulin's effects are generally associated with muscle and adipose tissue, leukocytes, fibroblasts, and mammary glands. Insulin does not directly affect brain and kidney tissue, intestinal mucosa, and red blood cells. Glucagon's effects are targeted mainly on the liver.

43.

A. B.

Increased levels ofglucagon and insulin. Increased levels of glucagon and decreased

C. D.

Decreased levels of glucagon. None of the above.

levels of insulin.

Symptoms reported by a patient include weakness. dizziness, confusion, and hunger. Furthermore, somr

tremors, palpitations, and nervousness are alsp reported. These last symptoms are characteristic d hypoglycemia. The development of these sympron$ is due to:

A. B. C. D. Copyright

@

by The Berkeley Review

3A4

abnormal increases in insulin secretion.

abnormal decreases in insulin secretion. abnormally high levels of plasma glucose. abnormally low levels of free fatty acids.

The Berkeley Specializing in MCAT

Biology 44.

Qlucose, Glucagon, & Insulin

Passage

YI

What pancreatic hormonal response is expected after a heavy protein intake?

A. B.

c. D.

Increased levels of insulin and glucagon. Increased levels ofinsulin and decreased levels of glucagon. Decreased levels ofinsulin and glucagon. Decreased levels of insulin and increased levels ofglucagon.

45. From the data reported on glucose utilization of tissue and responsiveness of tissue to insulin, it may

be determined that the main function of insulin secretion is:

A. B. C.

D.

46.

increasing glucose uptake in the brain. increasing glucose release from the liver. access and storage of glucose in cells of the peripheral tissues. increasing glucose loss in the kidney.

Patients diagnosed as having diabetes mellitus are said to be in "a state of starvation in the midst of plenty." This analogy refers to:

A. B. C. D.

Copyright

extracellular glucose excess. extracellularglucosedeficiency. decreased effects of insulin on intestinal mucosa uptake of glucose. deficiency of fatty acids in neural tissue.

@

by The Berkeley Review

305

The Berkeley Keview Specializing in MCAT Preparation

Biology Passage

,

VIII

Calcium, PTII, Calcitonin, Calcitriol

(Questions 47-54)

Calcitonin acts to inhibit osteoclast activity and reduce bone resorption. This hormone also inhibits talcium and

Calcium and phosphate both play an important role in

the mineralization of bone

in vertebratei. Calcium

phosphate reabsorption in the kidney and increases the excretion ofthese ions in the urine.

is

obtained from the diet and is largely absorbed at the level of the intestine. The intracellular concentration of calcium is about 10-7 mol/I- while the extraceliular concentration of calcium is about l0-3 mol/L. Blood calcium levels are primarily determined by bone metabolism and urinary

47.

excretion.

Calcium metabolism is regulated through the actions of parathyroid hormone (PTH), calcitonin, and vitamin D3 (choiecalciferol). PTH is synthesized and secreted by the parathyroid glands, usually located in the central region of the thyroid gland near the trachea. Calcitonin is synthesized and secreted from parafollicular (or C) cells located in the thyroid gland. Cholcalciferol is synthesized

Administration of PTH leads to changes in plasma calcium and phosphate concentrations. Whicli of the following graphs BEST represents these changes? A. J o

p o

0 O

must first be hydroxylated in the liver and

B. J

increased by PTH.

o tr

p

Bone is composed of an organic matrix consisting of collagen fibers and a ground substance composed of extracellular fluid and proteoglycans. The collagin fibers

o

o 6

help to give bone its great tensile strength, white tne ground substanca helps to control the deposition of

t-

0 0

Time (hours)

PTH added C. ..1

The collagen matrix and ground substance is laid down

o

o

E

by bone cells called osteoblasts. The tissue which is formed, called an osteoid, can enlrap some of the osteoblasts. Entrapped osteoblasts are called osteocytes.

Calcium

E

As the bone grows, calcium salts precipitate on the collagen fibers. Bone is also undergoing resorption by

0 fr

PTH causes calcium absorption from bone by

fc p o

Phosphate

(-)

cells called osteoclasts.

Time (hours)

PTH added

stimulating osteociastic activity and transiently inhibiting osteoblastic activity. At the level of the kidney, pTH increases calcium absorption in the distal tubules and collecting ducts and greatly decreases the reabsorption of phosphate at the proximal tubules.

D. J o

'6 p

6'

The activated form of cholecalciferol (1,25-(OH)Z-D/

o

()

has target receptors in the intestine, bone, and kidney (to

name but a few tissues). In the intestine this hormone promotes absorption of calcium and phosphate (following as the counterion), while in bone it promotes resorption of both calcium and phosphate. In the kidneys this hormone promotes the reabsorption of both calcium and phosphate so that little is excreted in the urine. by The Berkeley Review

B

U

calcium salts, like hydroxyapatite (Ca5@O4):OH). The calcium salts help to provide for the great compressional strength found in bone.

@

Time (hours)

PTH added

then hydroxylated in the kidney. The activity of the hydroxylase enzyme in the kidney is regulated and

Copyright

I

O

Q

animals from a photolytic reaction involving UV light and a sterol derivative. In order for this inactive hormone to be

it

o "o

in an inactive form in the skin of

activated,

Passage Vtr

0 G

Time (hours)

PTH added

506

The Berkeley Revieu Specializing in MCAT Preparation

Biology 48.

Calcium, PTII, Calcitonin, Calcitriol

Passage Vtr

49. Calcium and phosphate absorption in the intestines

Which of the following structures BEST represents vitamin D3?

is stimulated by an increase in:

A.

I. II. III.

1,25(OH)2D3 Calcitonin PTH

A.

I only I and II only III only I and III only

B. C. D.

50.

B.

In order for the secretion of calcitonin to have a greater effect on the concentration ofcalcium ions in the plasma, which of the following statements must

be true regarding osteoclast activity and plasma calcium levels?

A. B. C. D.

C.

51.

Increased osteoclast activity hypercalcemic plasma. Decreased osteoclast activity hypercalcemic plasma. Increased osteoclast activity hypocalcemic plasma. Decreased osteoclast activity hypocalcemic plasma.

coupled with

a

coupled with

a

coupled with

a

coupled with

a

Hypophosphatemic rickets is an Xlinked dominant trait that leads to decreased levels of phosphate reabsorption in the kidneys. Which pedigiee sho*n below BEST represents this disease?

A.

B.

':l1IT

rr+\-u+-l fl-rl [i J-I-J-I

D.

Jopyright

@

by The Berkeley Review

307

,,,

u,

C.

D.

The Berkeley Review Specializing in MCAT Preparation

Biology

Calcium, PTII, Calcitonin, Calcitriol

Passage Vtr

52. Familial

hypophospharemia is BEST treated by diet modification and supplying adequate amounts of:

A. B. C. D.

53.

calcium. phosphate. calcium and phosphate. phosphate anA t,ZS1Ofq2O3.

Hypoparathyroidism is BEST characrerized by:

I. Ir. III. IV.

increased increased increased increased

A.

I only II and III only Itr only I and fV only

B. C.

D.

54-

osteoblast activity. osteoclast activity.

neural excitability. plasma calcium concentrations.

In the graph shown below, all of the following statements concerning the relationship between PTH, calcitonin, and calcium are true EXCEpT:

o o c€

o o o o

o

o o

a

:-

5

Total Ca2+ concentration in piasma

A. B. C. D.

PTH is a hypercalcemic hormone. calcitonin is a hypocalcemic hormone.

a positive linear relationship exists between calcitonin secretion and the concentration of plasma calcium.

a positive linear relationship exists between PTH secretion and the concentration of plasma calcium.

Copyright

@

by The Berkeley Review

The Berkeley Kevier Specializing in MCAT

\

Biology

Erythroblastosis Fetalis

IX (Questions 55-61)

Passage

Erythroblastosis fetalis (EF)

56,

is also known

Which of the following statements is TRUE?

A.

as

hemolytic disease of the newborn. Hemolysis is rupture of red blood ceils, so in hemolytic disease, anemia is

B.

.ommon! due to the lysis of red blood cells. In the EF --ondition, maternal antigens cross the placental barrier, auack proteins on the surface of the red blood cells of the r-etus, and lyse the cells. A very specific set of conditions must exist for this disease to occur.

Passage IX

C. D.

The placenta allows passage of all blood products from the mother to the fetus. The Rh factor is nor a component of the ABO blood group system. The mother makes red blood cells for the fetus

in the placenta. Terminated pregnancies have no effect on future development of EF.

The mother must be Rhesus (Rh) factor negative, the :etus musf be Rh factor positive, and the mother's immune s\ stem must be sensitized to the Rh positive antigen

tlrough previous full-term pregnancy or abortion. The Rh :actor antigen is transmitted as a dominant trait, so that rnly people who are homozygous recessive are Rh factor

57.

What preventive measure couid protect subsequent fetuses il an Rh-negative mother gave birth to an

regative.

Rh-positive fetus?

Roughly 90Vo of the cases of EF result from ':nsitivities to the D antigen on the Rh factor. When Rh :ositive blood enters the circulation of an Rh negative lother, antibody formation against D may be induced. This exposure may be during an accidental infusion, iuring pregnancy, delivery, or during a miscarriage or "nortion. During a first pregnancy, there is usually little

A. B. C. D.

:rchange of fetal and maternal blood, except near the end -

i

Give the fetus a blood transfusion with

Rh_

negative blood.

Treat mother with a set of antibodies directed against the anti-Rh antibodies. Treat fetus with antibodies against the anti-Rh antibodies.

the pregnancy or during delivery.

This time frame does not allow for antibody formation by an attack on the fetal biood cells. The :ioblem lies in subsequent pregnancies. Small amounts of rtigen, even the amount in I mL of fetal blood entering .:e mother's circulation, promote rapid increases in hei .r:i-D antibody titer. IgG is produced, and it can easily :.oss the placental barrier into the fetal blood supply. ;','en in the ideal conditions for EF, sometimes the diieaie : res not manifest, due to variable physiological .

Give the mother a blood transfusion with Rhpositive biood.

--'ilowed

58. An infant wirh severe EF has jaundice, a yellow

coloring due to excess bilirubin, a breakdown product of heme. In what tissue or organ is heme degraded into bilirubin?

A. B. C. D.

::nditions.

Spleen Bone marrow

Liver

All of the above

59. An Rh-positive mother is pregnant with an Rhnegative fetus.

55, A

Rh-negative woman and a heterozygous Rhpositive man have one child together. The woman has never been pregnant before. What is the

-

A.

likelihood that the child wiil be born with EF?

B.

A. B. C. D.

C.

rpyright

l)AVo l5Vo

D.

50Vo

jVo

@

by The Berkeley Review

309

Will the fetus develop EF?

No, there are no maternal antigens to the Rh factor antigens. Yes, the mother can still make antigens to the Rh factor of the fetus. No, the fetal antibodies protect its red blood cells.

No, but fetal antibodies attack the maternal red blood cells.

The Berkeley Review Specializing in MCAT preparation

Biology

Erythroblastosis Fetalis

Passage IX

60. What variables could affect the severity of hemolysis in an Rh-positive fetus whose mother is Rh-negative?

I. II. Iil.

Amount of blood transferred. Sensitivity of mother to D antigen. Number of pregnancies.

A. I only B. I and II only C. II and III only D. I,II, and IIi

61. Which

of the following clinical signs could

be

consistent with a diagnosis of EF in a newborn?

I. il. ilI.

High levels of bilirubin in the blood. Low levels of hemoglobin in the blood. Increased levels of erythrocytes.

A. B. C. D.

I only I and II only II and III only I, II, and III

Copyright

@

by The Berkeley Review

5ro

The Berkeley Review Specializing in MCAT Preparation

Biology

Septic Shock

Passage X (Questions 62-67)

64. A

defect

in which of the following cells

would

inhibit the production of soluble anribodies?

Septic shock, a disease characterized by hemodynamic derangements and multi-organ malfunction, has generally been associated with a gram-negative infection. However,

A. B. C. D.

it is becomming increasingly clear that gram positive organisms are equally responsible for sepsis. Many of these gram positive ogranisms release molecules known as superantigens. These superantigens can induce

Passage X

Mast cells

Cytotoxic T cells Plasma cells

Erythrocytes

T cell

proliferation without regard to the antigenic specificity of ihe cell.

65. In

A

common sign of septic shock is widespread rctivation of coagulation leading to widespread rntravascular clotting. Microbial products activate Factor -\I[, a molecule involved in b]ood clotting. Activation of :ris factor initiates the intrinsic coagulation pathway and .:so the bradykinin pathway. Bradykinin is a potent ',

would expect to see

A. B. C. D.

asodilator and also increases the permeability of vascular

:ndothelial cells. Cytokines, such as interleukin-1 and tumor necrosis :.;tor, activate tissue factor III. This factor is found on the - iler membrane of macrophage and endothelial cells, and ,

an experiment, a sepsis patient is treated with XII antibodies. After treatment. one

anti-factor

a:

total lack of intravascular clotting. rise in the level of intravascular clotting. rise in the patient's blood pressure. decrease in the patient's blood pressure.

66. Bradykinin

acts to increase the radius of a given blood vessel by a factor of 2. The flow of blood

::mulates the extrinsic coagulatory pathway.

through the vessel should increase by a factor of:

A. B. c. D.

r,l- A Gram-positive cell differs from a Gram-negative

2. 4. 8. 16.

cell in that a Gram-positive cell:

-{.

does not have an outer membrane on its cell

B. C.

does have an outer membrane on its cell wall. contains a thin peptidoglycan layer adjacent to the plasma membrane. contains no peptidoglycan layer adjacent to the plasma membrane.

D.

67.

wall.

The activation of bradykinin most likely results in:

A. hypotension in the patient. B. cytokine release. C. macrophageactivation. D. stimulation of the extrinsic

coagulation

pathway.

\lacrophages destroy microorganisms through:

A. B. C. D.

r'

:.qht

exocytosis, secreting toxins which eventually form membrane attack complexes. xxocytosis, surrounding the foreign particle with a lipid bilayer which protects the host organism. endocytosis, engulfing foreign particles which eventually will fuse with a lysosome. endocytosis, engulfing foreign particles which eventually will fuse with a peroxisome.

@

by The Berkeley Review

5l I

The Berkeley Review Specializing in MCAT Preparation

Biology Passage

Calcitonin and Osteoporosis

XI (Questions 68-75)

71.

Calcitonin (CT) is a polypeptide hormone secreted by the parafollicular cells of the thyroid gland in mammals.

32 amino acids make up the hormone, and a disulfide

CT works as an antagonist of parathyroid hormone (PTH) In response to small increases in plasma calcium, CT is released and acts on the kidnw and bone to decrease the calcium level. In the bone *airix, osteoblasts synthesize bone, and osteoclasts catabolize bone. The

choices is a probable explanation why salmon CT is 30 times more active in humans than human CT?

I. II.

Calcitonin is used pharmacologicaily as a treatment for osteoporosis. Saimon CT is commonly used. Although salmon CT differs markedly from human CT, it is about 30 times more potent when used in non-ailergic humans to treat osteoporosis. Treatment with CT is not without its own side effects. CT treatment for osteoporosis increases

IfI. A. B. C. D.

supplementation to avoid hyperparathyroidism.

Osteoporosis is a disorder ofbone characterized by a decrease in bone quantity, most common in women

Salmon CT attaches more tightly to the CT

I only I and III only II and III only I, II, and III

73. If

a person begins a calcium supplement regimen and doubles calcium intake, whit would be the

following menopause and in elderly men and women. Which of the following conditions would lead to the GREATEST decrease in bone quantity?

response in CT secretion?

A. B. C. D.

Decreased osteoblast activity, decreased osteoclast activity.

Increased osteoblast activity, decreased

B.

Salmon CT is more resistant to degradation by human enzymes. Salmon CT attaches more strongly to the DNA of the osteoclasts. receptor.

plasma PTH and requires simultaneous calcium

A.

Methionine-threonine Methionine-methionine Cysteine-cysteine Cystine-cysrine

72. Which of the following

main eff-ects of CT are ( i ) inhibition of osteoclasts and (2) a transient increase in urinary calcium and phosphate.

58.

Which two amino acids form a disulfide bridse?

A. B. C. D.

bridge links residues 1 and 7. The entire CT molecule and the. disulfide bridge are required for full biological activity.

Passage XI

CT CT CT CT

secretion secretion secretion secretion

increases. decreases.

remains unchansed. halts completelyl

osteoclast activity. C.

Decreased osteoblast activity, increased

D.

Increased osteoblast activity, increased

osteoclast activity.

74.

osteoclast activity.

69.

What is the most abundant mineral in the human

A. Bn B. Water C. Calcium D. Zinc

75.

Ingestion to avoid allergic reaction.

Injection to avoid hydrolysis.

C.

Ingestion to avoid hydroiysis. Injection to avoid allergic reaction.

What are the symptoms of an allergic reaction it" foreign protein in the bioodstream?

Based on the passage, what is the role of pTH?

A. B. C. D.

A. I only B. I and II only C. III only D. I, II, and III

Copyright

@

by The Berkeley Review

312

polypepride, administered?

B.

I. II. IU.

PTH increases plasma calcium. PTH decreases plasma calcium. PTH increases urinary calcium. PTH increases urinary phosphate.

a

A.

D.

body?

70.

How is salmon CT,

Flushing or reddening ofthe skin. Skin welrs (hives).

Difficultybreathing.

The Berkeley Rerieu Specializing in MCAT preparatim

Biology Passage

XII

Vertebrate Immune System

(Questions 76-83)

77.

When certain types of antibodies bind to their tarset

cells, they attract a series of proteins coiiectivJly

In order to protect vertebrates from infection, the im,mune system has evolved both an antibody-based and a cell-mediated response to foreign antigens. B cells, which

known as complement. These pioteins can then form pores which allow small molecules to freely diffuse across the plasma membrane. What effect would this have on a target cell?

originate and develop in hemopoietic tissues (bone

marrow and fetal liver), are responsible for producing and secreting antibodies which bind to antigen particl6s. T celis also originate in hemopoietic tissues but later migrate to and mature in the thymus during early development. Cytotoxic T celis are mainly .esponsible for mounting a cell-mediated defense by directlyiausing the death of infected celis. While B cells can be activat; by the binding of extracellular antigen to special receptors on the plasma membrane, T cells must come in direcicontact r.r,ith

Passage XII

A.

The ceil would die due to its inability to

initiate action potentials. The celi would become hyperpolarized.

B. C.

The cell would lyse due- to an upset water

D.

The cell would shrink due to an upset water

balance. balance.

infected ceils in order to become activated.

How the immune system differentiates between self and foreign antigens has been the topic of much study. Immunologists in the first half of thii century p.opor"d two main theories: Theory

78. Which of the foliowing

I

consistent with

Vertebrates inherit genes that encode receptors ipresent on the surface of B and T cells) that are capable

of binding oniy foreign antigens. The immune

doesn't react against host tissues because lacks the receptors which bind selfantigens.

it

be

L A foreign cell line is injected into a mouse

embryo. Further injections of the cell line into

system

the adult mouse do not elicit an immune

geneiically

response.

il. Transplantation of organs

between monozygotic twins does not result in orsan

Theory 2

The.. immune system

is inherently

ilI.

capable of responding to both self and foreign antigens, but it becomes "tolerant" to self antigens during early

rejection by the immune system.

Cells transplanted between mice which

are

genetically identical (i.e., from the same inbred strain) are tolerated by the new host's immune

development. Since foreign antigens aren't preseni during embryonic stages, the immune system does not deveiop i tolerance to them.

76.

statements would BOTH Theory 1 and Theory 2?

system.

A. B. C. D.

I oniy II only II and III only I, II, and III

According to the passage, removal of the thymus from an adult human would most likely result in:

A. drastically B.

decreased antibody-mediated

lmmune fesponse. drastically decreased cell,mediated immune

79.

response.

C. drastically D.

Copyright

decreased antibody and cell_ mediated immune response. littie or no change in the effectiveness of either type of immune response.

@

by The Berkeley Revierv

Tolerance to self antigens breaks down in the human autoimmune disease myasthenia gravis, resulting in the production of 4ntibodies against the patient's skeletal muscle acetylcholine receptors. Wt ictr of the foliowing is a likely symprom of this disease?

A. B. C. D. .'t^t

Irregularities in heart contraction. Weakness and difficulty breathing. Paralysis of the gastrointestinal tract. Dementia.

The Berkeley Review Specializing in MCAT preparation

Biology 80.

Vertebrate Immune System 83.

Recently activated B and T cells are examined via electron microscopy. Which of the following would be the most likely observations?

A. The B cells have a

mitochondria than the T cells.

B

B.

The

C. D.

endoplasmic reticulum than the T cells. Both the T and B cells lack nuclei. No differences between the two types of cells are revealed at the level of electron

Virally infected cells are usually killed by cytotoxic T cells. The T cells can most likely target:

A. B. C. D.

greater number of

cells have considerably more rough

Passage XII

most types of cells in the body. only cells lining the blood vessels.

epithelial cells only. other blood cells onlv.

microscopy.

81.. A sample of B cells is removed from an adult mouse. A highly radioactive antigen X is added to the B cells, killing the few that bind strongly (
View more...

Comments

Copyright ©2017 KUPDF Inc.
SUPPORT KUPDF